Вы находитесь на странице: 1из 152

ЗАЙЦЕВ АРТЁМ

СПРАВОЧНАЯ КНИГА ПО
МАТЕМАТИКЕ
КУРС СРЕДНЕЙ И СТАРШЕЙ ШКОЛЫ

 Эффективная подготовка к государственным


экзаменам (ОГЭ, ЕГЭ).
 Адресовано обучающимся общеобразователь-
ных учебных заведений, абитуриентам.
 В пособии подробно разобраны примеры задач из
ОГЭ и ЕГЭ, которые часто вызывают у обучающих-
ся затруднения при подготовке к экзаменам

ПОСОБИЕ СОДЕРЖИТ ВСЕ БАЗОВЫЕ


ОПРЕДЕЛЕНИЯ И ФОРМУЛЫ ШКОЛЬНОГО
КУРСА МАТЕМАТИКИ
ЗАЙЦЕВ АРТЁМ

СПРАВОЧНАЯ КНИГА ПО
МАТЕМАТИКЕ
КУРС СРЕДНЕЙ И СТАРШЕЙ ШКОЛЫ

Симферополь, 2023
Автор: Зайцев Артём Сергеевич

З-17
Справочная книга по математике. 5-11 классы: учебное посо-
бие для общеобразовательных учреждений / [А.С. Зайцев], – издание 2023
– 150 с.
Учебное пособие состоит из двух разделов («Алгебра», «Геомет-
рия») и приложения. Цель книги – собрать и сгруппировать темы школь-
ного курса математики, а также помочь обучающимся после изучения тем
применять полученные знания на практике при решении задач. С данной
справочной книгой можно работать независимо от того, по каким учебни-
кам школьники учились в предыдущие годы.
В справочной книге используются для наглядности доступные
таблицы и схемы, а также представлен весь учебный материал по предме-
ту, проверяемый на ОГЭ и ЕГЭ: «Математика 5-6 классы», «Алгебра 7-9
классы», «Алгебра и начала анализа 10-11 классы», «Теория вероятностей
и статистика 7-9 классы», «Геометрия 7-11 классы».
В приложении к данной книге можно найти некоторые типы задач
и их подробные решения (в том числе и авторские) из ОГЭ и ЕГЭ, которые
часто вызывают трудности у обучающихся при подготовке к экзаменам.
Учебник также нацелен на помощь обучающимся в подготовке к
поступлению в вузы и сдаче вступительных испытаний.
АЛГЕБРА
Алгебра (от араб. аль-джабр «восполнение») – раздел мате-
матики, который можно нестрого охарактеризовать как обобщение и рас-
ширение арифметики; в этом разделе числа и другие математические объ-
екты обозначаются буквами и другими символами, что позволяет записы-
вать и исследовать их свойства в самом общем виде.

ТЕМА 1. НАТУРАЛЬНЫЕ ЧИСЛА И ДЕЙСТВИЯ НАД НИМИ


Натуральные числа (НЧ) – это те числа, которые используются
при счёте предметов: Множество натуральных чисел обо-
значается символом .
Сравнение натуральных чисел
Из двух НЧ большим (меньшим) является то число, которое на
числовом луче стоит дальше (ближе) от нуля.

0 1 2 3 4 5 6 7 8 9 10 11 12 …
Простые и составные числа
Простые числа – натуральные числа, которые имеют два разных
делителя (1 и само число).
Примеры: 2, 3, 5, 7, 11, 13, 17, 19, 23, 29, 31, … – простые числа.
Составные числа – натуральные числа, которые имеют более
двух делителей.
Примеры: 4, 6, 8, 9, 10, 12, 14, 15, 16, 18, 20, … – составные числа.
Наибольший общий делитель и наименьшее общее кратное
Наибольшим общим делителем (НОД) чисел и называется
наибольшее число, на которое делится и число , и число .
Наименьшим общим кратным (НОК) чисел и наз. наимень-
ше число, которое делится и на число , и на число .
Примеры: НОД (5;15) = 5; НОК (5; 15) = 15, НОК (15; 9) = 45.
Формула для нахождения НОК: ( )
( )
( ) ( ) .
Далее рассмотрим признаки делимости чисел.

3
Признаки делимости чисел
( ) если его последняя цифра делится на ( );
( ) если сумма его цифр делится на ( );
если число, состоящее из двух последних цифр, де-
лится на ;
если оно делится на и на одновременно;
если оно четное и его цифровой корень делится на ;
Число можно если три его последние цифры – нули или образуют
разделить на число, которое делится на ;
если его последняя цифра – ;
если сумма цифр, стоящих на нечётных местах,
либо равна сумме цифр, стоящих на чётных местах,
либо отличается от неё на число, делящееся на ;
если число, состоящее из двух последних цифр, де-
лится на .
Цифровой корень числа – это цифра, которая получается в ре-
зультате сложения всех цифр некоторого числа, затем всех цифр найден-
ной суммы и так до тех пор, пока не останется одна цифра. Цифровой ко-
рень для числа, записанного в десятичной системе счисления, равен остат-
ку от деления его на .

ТЕМА 2. ОБЫКНОВЕННЫЕ ДРОБИ, ДЕСЯТИЧНЫЕ ДРОБИ,


СМЕШАННЫЕ ЧИСЛА И ДЕЙСТВИЯ НАД НИМИ

Обыкновенной дробью называется выражение , где ,


. Число называется числителем, а число – знаменателем. Дробная
черта означает деление. Знаменатель дроби показывает, на сколько равных
частей делится число (величина), числитель – сколько таких частей взято.
Дробь называется правильной, если её числитель меньше знаме-
нателя ( ). Дробь называется неправильной, если ее числитель равен
знаменателю или больше его ( ).
Смешанным числом называется сумма натурального числа и
правильной дроби, записанная без знака «+».
Примеры: ; .

4
Выделение целой части из неправильной дроби и наоборот
Чтобы из неправильной дроби выделить целую часть, необходимо
разделить с остатком числитель на знаменатель: неполное частное будет
целой частью, остаток – числителем, а знаменатель – тот же.
Примеры: = ; =4 .
Чтобы записать смешанное число в виде неправильной дроби,
необходимо умножить его целую часть на знаменатель дроби и прибавить
числитель, полученное число – это числитель неправильной дроби, а зна-
менатель остается прежним.
Пример: = = .
Основное свойство дробей: если числитель и знаменатель дроби
умножить или разделить на одно и то же число, отличное от нуля, то, по-
лучим дробь, которая равна данной.
Пример: = = .
Сокращение дроби – это деление числителя и знаменателя на
общий делитель числителя и знаменателя дроби, который больше едини-
цы. Примеры: = ; = .

Сложение обыкновенных дробей и смешанных чисел


1) Сложение дробей с одинаковыми знаменателями: .
Пример:
2) Сложение дробей с разными знаменателями: , если
НОД (b; d) = 1.
Пример: .
3) Сложение смешанных чисел: .
Пример: .

Вычитание обыкновенных дробей и смешанных чисел


1) Вычитание дробей с одинаковыми знаменателями: .
Пример: .

5
2) Отнимание дробей с разными знаменателями: , если
НОД (b; d) = 1.
Пример: .
3) Отнимание смешанных чисел: .
Пример: .

Умножение обыкновенных дробей


1) Умножение дробей: .
Пример: .
2) Умножение смешанных чисел: .
Примеры: .

Деление обыкновенных дробей


1) Чтобы разделить дробь на дробь необходимо заменить делание
умножением, а вторую дробь заменить обратной (т.е. перевернуть
ее): .
Пример: .
2) Деление смешанных чисел: .
Пример: .

Умножение и деление десятичных дробей


Умножение десятичных дробей происходит в три этапа:
1. Десятичные дроби записывают в столбик и умножают как
обыкновенные числа.
2. Считаем количество знаков после запятой у первой десятичной
дроби и у второй. Их количество складываем.
3. В полученном результате отсчитываем справа налево столько
же цифр, сколько получилось их в пункте выше, и ставим запятую.
Деление десятичной дроби на натуральное число
1. Делим десятичную дробь на натуральное число по правилам де-
ления в столбик, не обращая внимания на запятую.

6
2. Ставим в частном запятую, ко-
гда заканчивается деление целой части
делимого.
Запомните! Если целая часть де-
лимого меньше делителя, то в частном
ставим 0 целых.
Деление натурального числа на десятичную дробь
1. Считаем количество знаков справа от запятой в десятичной
дроби.
2. Умножаем и делимое, и делитель на 10, 100 или 1000 и т.д., что-
бы превратить десятичную дробь в целое число.
3. Делим числа как натуральные.
Запомните! Чтобы разделить (умножить) десятичную дробь на 10,
100, 1000 и т.д., нужно перенести запятую в этой дроби на столько цифр
влево (вправо), сколько нулей стоит после единицы в делителе (множите-
ле).

ТЕМА 3. ЦЕЛЫЕ И РАЦИОНАЛЬНЫЕ ЧИСЛА, ДЕЙСТВИЯ


НАД НИМИ. ПРОПОРЦИИ
Целые числа (ЦЧ) – это натуральные числа, им противополож-
ные числа и число 0. Множество ЦЧ обозначается символом .

Представление целого числа в виде дроби


Любое целое число a можно представить в виде дроби: где
. Например, число

Сравнение положительных и отрицательных чисел


Любое отрицательное число меньше нуля и любого положитель-
ного числа, а нуль меньше любого положительного числа.
Пример: –8 < 0; –7 < 3; 0 < 5.
Сложение отрицательных чисел и чисел с разными знаками
1) Сложение отрицательных чисел. Чтобы сложить два отрица-
тельных числа, необходимо их прибавить и поставить их общий знак:
( ) ( ) где и – положительные числа.

7
Пример: (– ) ( ) .
2) Сложение чисел с разными знаками. Чтобы сложить два числа с
разными знаками, необходимо из большего числа по модулю вычесть
меньшее число по модулю и поставить знак большего числа по модулю.
Пример: (| | | |) .
Вычитание положительных и отрицательных чисел
Отнять от числа число означает прибавить к числу число,
противоположное , т.е.: ( )
Пример: ( ) .
Умножение положительных и отрицательных чисел
1) ( ) ( ), где – положительные числа.
Пример: –5 4 = –20.
2) ( ) , где – положительные числа.
Пример: ( ) .
Деление положительных и отрицательных чисел
1) ( ) ( ), где – положительные
числа.
Пример: ( )
2) ( ) , где – положительные числа.
Пример: ( )
Рациональные числа (РЧ) – это числа, которые можно предста-
вить в виде , где . Множество РЧ обозначают символом .
Примеры РЧ: .
Любое рациональное число – неоконченная (бесконечная) перио-
дическая десятичная дробь.
Также существуют иррациональные числа – это бесконечные
непериодические дроби. Обозначаются такие числа символом .
Примеры: √ √ √
Иррациональные и рациональные числа вместе образуют веще-
ственные (действительные) числа. Обозначаются эти числа символом .
Примеры: √ √

8
Свойства деления

НА НУЛЬ ДЕЛИТЬ НЕЛЬЗЯ


Пропорции
Пропорции: или где – крайние члены,
– средние члены. Произведение крайних членов равно произведению
средних членов:

ТЕМА 4. ПРОЦЕНТЫ. ФОРМУЛА ПРОИЗВОДИТЕЛЬНОСТИ В


ЗАДАЧАХ
Сотую часть любой величины, или числа называют процентом.
Процент обозначают символом « ». .
Чтобы превратить десятичную дробь в проценты, нужно её умно-
жить на , а чтобы превратить проценты в десятичную дробь, нужно
число процентов разделить на .
Пример:
Производительность – это объём работы, выполняемый за еди-
ницу времени.
Большинство задач на работу сводятся к применению формулы
производительности ( – производительность, – работа, – время):

При совместной работе производительности складываются.

ТЕМА 5. СТЕПЕНЬ С НАТУРАЛЬНЫМ И ЦЕЛЫМ ПОКАЗА-


ТЕЛЕМ
Степень числа с натуральным показателем , большего едини-
цы, называют произведением множителей, каждый из которых равен
.
раз
Первой степенью числа называют само число: .
В записи число называется основанием степени, – по-
казателем степени, – степенью, – значением степени.

9
Свойства степеней:
1)

2) или

3) ( )
4) ( )

5) ( )

6) ( ) ( ) .

7) – не определён.

8) . Выражение , где – не определено.

Стандартный вид числа


Стандартный вид числа: , где и Число
называют порядком числа.

ТЕМА 6. ОДНОЧЛЕНЫ И МНОГОЧЛЕНЫ


Одночленом называется выражение, которое содержит числа,
степени переменных и их произведения. Число называют нулевым одно-
членом, и он не имеет степени.
Примеры:
Одночлен в стандартном виде – одночлен, который содержит
только один числовой множитель (коэффициент), стоящий на первом ме-
сте, и степени с разными буквенными основаниями.
Примеры: .
Степенью одночлена называют сумму показателей степеней всех
входящих в него переменных.
Пример одночленов третьей степени:
Многочленом называется алгебраическая сумма нескольких од-
ночленов.
Пример: .
Одночлены, из которых состоит многочлен, называют его члена-
ми. Многочлен, который содержит два или три слагаемых, называют соот-
ветственно двучленом и трёхчленом.

10
Подобные члены многочлена – это одночлены, запись которых в
стандартном виде отличается лишь коэффициентами (т.е. буквенная часть
полностью одинаковая).
Приведение подобных членов – это упрощение многочлена, при
котором алгебраическая сумма подобных членов заменяется одним членом
(т.е. складываем или вычитаем числа при одинаковых переменных).
Стандартный вид многочлена – это запись многочлена, все чле-
ны которого имеют стандартный вид, и среди них нет подобных.
Пример:
Степенью многочлена стандартного вида называется максималь-
ная (наибольшая) степень переменной, в него входящей.
Пример многочленов третьей степени:
При вычитании многочленов пользуются правилом раскрытия
скобок: если перед скобками стоит знак «–», то скобки можно опустить,
заменив знак каждого одночлена, находящегося в скобках, на противопо-
ложный.
Пример: ( ) ( )
Умножаются многочлены по принципу
. Такой принцип ещё называют «правилом фонтанчика».
Пример: ( ) ( )

Разложение многочлена на множители – тождественное преоб-


разование, превращающее сумму в произведение нескольких множителей
(каждый множитель может быть как многочленом, так и одночленом).
Для того чтобы разложить многочлен на множители можно выне-
сти общий множитель за скобку или воспользоваться формулой сокращён-
ного умножения (зависит от ситуации).
Формулы сокращённого умножения
1) ( ) – квадрат суммы (разности).
2) ( )( ) – разность квадратов.
3) ( )( ) – сумма (разность) кубов.
4) ( ) – куб суммы (разности).

11
ТЕМА 7. ЦЕЛЫЕ И ДРОБНЫЕ ВЫРАЖЕНИЯ. АЛГЕБРАИЧЕ-
СКИЕ ДРОБИ
Целое выражение – это математическое выражение, составленное
из чисел и буквенных переменных с помощью действий сложения, вычи-
тания и умножения.
Пример: ;
Выражение называется дробным, если у него присутствует деле-
ние на переменную (выражение, содержащее переменную).
Пример: ;
Аналогичным образом определяются целые и дробные уравнения.
Алгебраической называется дробь, если числитель и знаменатель
этой дроби являются алгебраическими выражениями (многочленами).
При умножении числителя и знаменателя дроби на одно и то же
алгебраическое выражение, получаем дробь, которая равносильна исход-
ной. Используя данное свойство дроби, можно сокращать алгебраические
дроби на общий множитель числителя и знаменателя.
( ) ( )( ) ( )
Пример: ( ) ( ( ) (
.
( ) ) )

ТЕМА 8. АРИФМЕТИЧЕСКИЙ КВАДРАТНЫЙ КОРЕНЬ И ЕГО


СВОЙСТВА
Квадратным корнем из числа называют число , квадрат кото-
рого равен (т.е. ).
Арифметическим квадратным корнем из числа называется
неотрицательное число, квадрат которого равен . Арифметический квад-
ратный корень из числа обозначается знаком √ , где называется под-
коренным выражением, √ – знак радикала.
√ | |
Пример: √ √ √( ) | |
Действие, при помощи которого находится арифметический квад-
ратный корень, называется извлечением квадратного корня.

12
Свойства арифметических квадратных корней
1) √ √ √

2) √

3) При : √ √
При : √ √
4) При : √ √ .
При : √ √

ТЕМА 9. УРАВНЕНИЯ, НЕРАВЕНСТВА И ИХ СИСТЕМЫ


Уравнение – равенство, содержащее переменную (неизвестную).
Пример: – это уравнение, а – это уже выражение,
т.к. отсутствует равенство.
Решение уравнения (корень) – значение переменной, при кото-
ром уравнение превращается в верное равенство.
Решить уравнение – это значит найти все его корни или доказать,
что их нет.
Равносильные уравнения – это уравнения, которые имеют одни
и те же корни или их не имеют.
Линейное уравнение
Уравнение вида , где – переменная, и – некоторые
числа, называется линейным уравнением. Корень этого уравнения:

Основные свойства уравнений:


1. В любой части уравнения можно привести подобные слагаемые.
2. Если из одной части уравнения перенести слагаемые в другую
часть и изменить при этом знаки слагаемых на противоположные, получим
уравнение, равносильное данному.
3. При умножении (делении) обеих частей уравнения на одно и
тоже число, не равное нулю, получим уравнение, равносильное данному.
Уравнение, которое содержит две переменные (неизвестные),
называется уравнением с двумя переменными.
Пример: .

13
Решением уравнения с двумя переменными называют пару ( и )
значений переменных, которые превращают это уравнение в правильное
числовое равенство.
Линейное уравнение с двумя переменными
Уравнение вида , где – переменные, – неко-
торые числа, называется линейным уравнением с двумя переменными.
Если меньше или больше , то записывают так: или
. Такое выражение называется неравенством.
Знаки неравенств
– больше; – больше или равно;
– меньше; – меньше или равно.
Знаки « » и « » являются знаками строгого неравенства; знаки
« » и « » – знаки нестрогого неравенства.
Если обе части правильного неравенства умножить (разделить) на
одно и то же отрицательное число и изменить знак неравенства на про-
тивоположный, то получим правильное неравенство.
Пример: , разделив это неравенство на – , получим:
– – ( ) ( )
Если неравенство строгое, то ответ записываем в круглых скобках
«(» и «)».
Если неравенство нестрогое, то ответ записываем в квадратных
скобках « » и « ».
Не зависимо от того, какое неравенство (строгое или нестрогое) и
после знака, и перед знаком бесконечности всегда ставится круглая скобка.
Знак бесконечности записывается символом « ».
Бесконечность может быть как отрицательной, так и положитель-
ной: ( ).
Линейным называется неравенство вида (
), где и – числа, – переменная.
Решениями неравенства с одной переменой называется множе-
ство таких значений переменной, которые преобразуют её в верное число-
вое неравенство.

14
Системы уравнений с двумя переменными
Системой уравнений называются два или несколько уравнений, у
которых необходимо найти все общие решения. Уравнения системы запи-
сываются столбиком и объединяются фигурной скобкой.
Система уравнений называется линейной, если все уравнения,
входящие в систему, являются линейными. Систему двух линейных урав-
нений с двумя переменными обычно записывают в виде:
{
Решить систему уравнений означает найти все её решения или
доказать, что решений нет.
Если система имеет конечное число решений, то она называется
определенной. Если система имеет бесконечное число решений, то она
называется неопределенной.
Две системы называются равносильными, если они имеют оди-
наковые множества решений.
Способ подстановки для решения систем уравнений
1. Выразить одну переменную через другую из одного уравнения
системы (более простого).
2. Подставить полученное выражение вместо этой переменной в
другое уравнение системы.
3. Решить полученное уравнение и найти одну из переменных.
4. Подставить найденную в третьем шаге переменную в любое из
первоначальных уравнений системы и найти вторую переменную.
5. Записать ответ в виде пар значений ( ), которые были найде-
ны соответственно на третьем и четвёртом шагах.
Пример: Решите систему уравнений {
Решение:
1) Выразим переменную из первого уравнения системы:
{
2) Подставим выраженную переменную во второе уравнение
системы:

15
{
( )
3) Решим второе уравнение системы, раскроем скобки и полу-
чим:

4) Подставим полученное значение переменной y в первое урав-


нение системы:
{ {
5) Запишем ответ.
Ответ: ( )
Обратите внимание, что сначала указывается ответ x, а потом – y.
Графический способ для решения систем уравнений
Для решения системы графическим способом строят графики всех
уравнений, входящих в систему. Координаты точек пересечения графиков
являются решениями этой системы.
Способ сложения для решения систем уравнений
1. Уравнять модули коэффициентов при одной из неизвестных
(причём, сами числа возле этой переменной должны оказаться противопо-
ложными, если мы хотим сложить эти два уравнения).
2. Сложить уравнения почленно и решить полученное уравнение с
одной переменной, найти неизвестную.
3. Подставить найденное на втором шаге значение переменной в
одно из уравнений исходной системы, найти второе неизвестное.
4. Записать ответ.
Пример: Решите систему уравнений: {
Решение:
{
( ) ( )

16
Переменную находим путем подстановки найденого в любое
из уравнений исходной системы.

( )

Системы линейных неравенств с одной переменной


Два или более неравенства с одной переменной, относительно ко-
торых поставлена задача найти все общие решения, называют системой
неравенств с одной переменной.
Решениями системы неравенств называются такие значения пере-
менной, которые являются решениями сразу всех неравенств, входящих в
данную систему.
Решить систему неравенств – означает найти все её решения или
доказать, что решений нет.

ТЕМА 10. ФУНКЦИИ И ИХ ОСНОВНЫЕ СВОЙСТВА


Зависимости одной переменной от другой называются функцио-
нальными зависимостями.
Если каждому значению независимой переменной соответствует
единственное значение зависимой переменной, то такая зависимость назы-
вается функциональной зависимостью, или функцией. Например, у
каждого человека есть размер обуви. Людей очень много, а размеров обу-
ви мало. Если с одной стороны отметить всех людей (например, в виде
точек), а с другой – существующие размеры обуви, то можно от каждой
точки провести стрелку к одному из размеров обуви (в математике исполь-
зуют такую терминологию: можно отобразить множество людей на мно-
жество размеров обуви). Или, говоря по-другому, между этими множе-
ствами можно установить соответствие. Такие соответствия, отмеченные
линиями, и называются функциями.

17
Функцией называется зависимость переменной от переменной
, причём каждому значению соответствует единственное значение .
Функция обозначается или одной буквой или ( ), или равен-
ством ( ) где – независимая переменная или аргумент, – зави-
симая переменная или значение функции; ( ) – значение функции в
точке .
Система координат – это две взаимно перпендикулярные коор-
динатные прямые, пересекающиеся в точке, которая является началом от-
счёта для каждой из них. Ось абсцисс « » — горизонтальная ось, ось
ординат « » — вертикальная ось.
Координатная плоскость – это плоскость, в которой построена
система координат. Обозначается плоскость как « ». Координатная
плоскость «разбита» на 4 четверти.
Область определения функции ( ( )) – множество значений,
которые принимает аргумент ( ).
Пример: ( ) , область определения ( ): – ,
т.е. переменная – это любое число, кроме .
Область (множество) значений функции ( ( )) –множество тех,
которые принимает сама функция ( ) при всех значениях аргумента из
области определения.
Пример: ( ) , область значений
( ): – это любое число, кроме . Это число
мы не сможем получить никогда, т.к. в числителе
уже стоит число 2. Вспомним правило: чтобы
дробь была равна нулю числитель этой дроби дол-
жен быть равен нулю, а знаменатель – не должен
равняться нулю (график данной функции – гипер-
бола).

18
Рисунок к примерам области определения и области значений функции
Графиком функции ( ) называется множество всех точек
плоскости с координатами ( ), абсциссы которых равны значениям ар-
гумента, а ординаты – соответствующим значениям функции в точке .
Способы задания функции
 Аналитический (формульный) способ: функция задаётся с по-
мощью математической формулы.
 Табличный способ: функция задаётся с помощью таблицы.
 Описательный способ: функция задаётся словесным описанием.
 Графический способ: функция задаётся с помощью графика.
Возрастание и убывание функции
Функция ( ) является возрастающей, если большему значе-
нию аргумента соответствует большее значение функции.
Функция ( ) является убывающей, если большему значе-
нию аргумента соответствует меньшее значение функции.
Периодичность функции
Функцию ( ) называют периодической с периодом ,
если для любого из области определения числа и – также при-
надлежит области определения и выполняется равенство:

19
( ) ( ) ( )

Пример периодической функции .

Парные и непарные функции


Функция ( ) является парной (чётной), если для любого
значения « » из ( ) и значения « » из ( ) выполняется следующее
равенство:
( ) ( )
График парной функции симметричен относительно оси , а
график непарной (нечётной) функции ( ( ) ( )) симметричный
относительно начала координат.

Пример парной, непарной и ни паркой, ни непарной функций

20
Графики некоторых функций и их основные свойства
1) Функция – прямая пропорциональность.
1. Область определения: .
2. Функция непарная.
3. Для функция возрастает при , убывает при .
4. Область значений: .
5. График – прямая, которая проходит через начало координат.

2) Функция ( ) – обратная пропорци-


ональность.
1. Область определения: ( ) ( ).
2. Функция непарная.
3. Если , функция убывает на промежутке ( ) и на про-
межутке ( ). Если функция возрастает на промежутке ( )
и на промежутке ( ).
4. Область значений: ( ) ( ).
5. График функции – гипербола.

21
3) Функция ( ) – квадратичная
функция.
1. Область определения: .
2. Функция парная.
3. Возрастание и убывание: если , функция убывает на про-
межутке ( , возрастает на промежутке ); если , функция
возрастает на промежутке ( , убывает на промежутке ).
4. Область значений:
 если то );
 если , то ( .
5. График функции – парабола.

22
4) Функция ( ).
1. Область определения: .
2. Функция непарная.
3. Для функция возрастает, если , убывает, если .
4. Область значений: .
5. График функции – кубическая парабола.

5) Функция | | – функция модуля.


1. Область определений: .
2. Функция парная.
3. Возрастание и убывание функции:
 на промежутке ( функция убы-
вает;
 на промежутке ) функция воз-
растает.
4. Область значений: ).
6) Функция √ – не имеет своего
отдельного названия, читаем как «функция ко-
рень из ».
1. Область определения: ).
2. Функция ни парная, ни непарная.
3. На промежутке ) функция возрастает.
4. Область значений: ).

23
ТЕМА 11. ЛИНЕЙНАЯ ФУНКЦИЯ И ЛИНЕЙНЫЕ УРАВНЕ-
НИЯ
Линейной функцией называют функцию вида , где и
– некоторые числа, – независимая переменная.
Основные свойства линейных функций
Возраста-
Пар-
Функция График D(y) E(y) ние/убыван
ность
ие

возрастает
на

ни пар-
ные, ни
непар-
ные
убывает на

Линейная

парная постоянная

непар-
зависит от
ная

24
Линейным уравнением с одной переменной называют уравнение
вида где – переменная, и – числа.
Если то уравнение не имеет корней.
Если то корнем уравнения будет любое число.
Если , то уравнение имеет только один корень: .
Прямая пропорциональность
Функция при называется прямой пропорциональ-
ностью, – угловой коэффициент.
Эта функция является частным случаем линейной функции
при . Поэтому её графиком является прямая, проходящая
через начало координат.
Если , то график функции расположен в I и III коор-
динатных углах, если , то график функции расположен во II и IV ко-
ординатных углах.

ТЕМА 12. КВАДРАТИЧНАЯ ФУНКЦИЯ


Квадратичной называют функцию вида ; где
– действительные (вещественные) числа, причем . Графиком
квадратичной функции является парабола.
Квадратное уравнение (полное)
Квадратным называют уравнение вида , где –
переменная, – действительные (вещественные) числа, причем .
Число называют первым (старшим) коэффициентом, – вторым
коэффициентом, – свободным членом. Квадратное уравнение, у которого
хотя бы один из коэффициентов ( или ) равен нулю, называют непол-
ным квадратным уравнением.
Неполное квадратное уравнение вида
Уравнение вида всегда имеет два корня: и
. Такие уравнения, как правило, решают разложением его левой
части на множители.
Неполное квадратное уравнение вида
Если то уравнение вида корней не имеет.

25
Если то уравнение вида имеет два корня:

√ и √ .

Если то уравнение вида имеет один корень:


.
Формула корней квадратного уравнения
Выражение называют дискриминантом квадратно-
го уравнения .
Если , то квадратное уравнение имеет два корня; если
– один корень (тогда говорят о кратности данного корня, т.е. 2 одинаковых
корня); если , то квадратное уравнение в корней не имеет.
Корни квадратного уравнения при находят по формуле:

Свойства коэффициентов квадратного уравнения:

Теорема Виета
Если и – корни квадратного трёхчлена , то вы-

полняются равенства: {

Разложение квадратного трёхчлена на множители


Если и – корни квадратного трёхчлена, то
выполняется равенство:
( – )( – )

Уравнение оси симметрии параболы:

26
Координаты вершины параболы, заданной квадратным уравне-
нием находятся по формуле:

( )

Абсциссу вершины параболы также можно вычислить по форму-


ле: – корни квадратного трёхчлена. Ординату ( )
можно вычислить подставив значение в уравнение функции вместо .
Свойства квадратичной функции:
– если , то ветви параболы направлены вверх, а сама
парабола находится ниже оси и пересекает её в двух точках;
– если , то ветви параболы направлены вверх, а сама
парабола соприкасается с осью в точке ;
– если , то ветви параболы направлены вверх, а сама
парабола находится над осью и не имеет с ней общих точек;
– если , то ветви параболы направлены вниз, сама па-
рабола находится над осью и пересекает её в двух точках;
– если , то ветви параболы направлены вниз, а сама
парабола соприкасается с осью в точке ;
– если , то ветви параболы направлены вниз, а сама
парабола находится ниже оси и не имеет с ней общих точек.

ТЕМА 13. РАЦИОНАЛЬНЫЕ УРАВНЕНИЯ И НЕРАВЕНСТВА


( )
Уравнение является равносильным системе уравнений
( )
( )
{
( )
Уравнение ( ) ( ) называется рациональным, если ( ) и
( ) – рациональные выражения (целые или дробные выражения без знака
корня).
Рациональные уравнения делятся на целые рациональные уравне-
ния и дробно-рациональные уравнения.
Целое уравнение – это уравнение, у которого правая и левая ча-
сти являются целыми выражениями. То есть, если в дроби нет деления на

27
переменную, тогда рациональное уравнение будет называться целым (ли-
нейным) уравнением.
Примеры:
Дробно-рациональное уравнение – рациональное (без знака кор-
ня) уравнение, в котором левая или правая части являются дробными вы-
ражениями (присутствует деление на переменную).
Примеры:
( )( )
Всегда при наличии переменной в знаменателе, прежде всего,
необходимо найти ОДЗ (область допустимых значений). В ОДЗ удобно
записывать отдельно все выражения знаменателя с переменными, не рав-
ные нулю (т.к. на нуль делить нельзя). Например, выражение стоит в
знаменателе, значит, . Это означает, что переменная
может быть любым числом, кроме 2. Заметим, что ОДЗ не всегда сказыва-
ется на ответе, но учитывать его нужно всегда!
Шаги решения рационального уравнения после нахождения
ОДЗ
1) Найти общий знаменатель всех дробей, которые входят в урав-
нение.
2) Заменить данное уравнение целым, умножив обе его части на
общий знаменатель.
3) Решить полученное целое уравнение.
4) Исключить из полученных корней те, которые превращают в
нуль общий знаменатель (сверяем с ОДЗ) и записать ответ.
Дробные неравенства
( ) ( ) ( )
 Неравенство равносильно системам: { и {
( ) ( ) ( )
( ) ( ) ( )
 Неравенство равносильно системам: { и {
( ) ( ) ( )
( ) ( ) ( )
 Неравенство равносильно системам: { и {
( ) ( ) ( )
( ) ( ) ( )
 Неравенство равносильно системам: { и {
( ) ( ) ( )

28
Решение рациональных неравенств методом интервалов
Метод интервалов используется для решения неравенств вида
( ) ( ) ( ) ( ) . Данный метод основывается на
том, что неразрывная на промежутке функция может изменять знак только
в тех точках, где её значение равно нулю (но может и не изменять знак).
Суть метода заключается в разложении неравенства на множители
(с учётом ОДЗ) и решением линейного уравнения для определения знаков
сомножителей, подробнее описано ниже по шагам.
Шаги решения неравенств методом интервалов
1) Находим область определения функции ( ).
2) Находим нули числителя (и знаменателя, если он есть), для это-
го числитель и знаменатель выражения в левой части неравенства прирав-
ниваем к нулю и решаем полученные уравнения.
3) Определяем точки, которые соответствуют найденным нулям и
отмечаем их черточками на оси координат.
4) Определяем знаки выражения ( ) из левой части решаемого
неравенства на каждом промежутке и проставляем их на графике.
5) Наносим штриховку над нужными участками графика, руковод-
ствуясь следующим правилом: в случае, если неравенство имеет
ки или изображается, штрихуются «минусовые» промежутки, если же
мы работаем с неравенством, имеющим знаки или , то выделяем
штриховкой участки, отмеченные знаком « ».
6) Выбираем нужный нам промежуток и записываем ответ.

ТЕМА 14. АРИФМЕТИЧЕСКАЯ И ГЕОМЕТРИЧЕСКАЯ ПРО-


ГРЕССИИ
Арифметической прогрессией называют последовательность
, каждый член которой, начиная со второго, равен предыду-
щему, к которому прибавляется одно и то же число , которое называют
разницей (разностью) арифметической прогрессии.
Пример: – арифметическая прогрессия,

Определяется -й член арифметической прогрессии по формуле:


( )

29
где – -й член, – первый член, – разница прогрессии.
Каждый член арифметической прогрессии, начиная со второго,
равен среднему арифметическому двух соседних членов:

Сумма первых членов арифметической прогрессии равна сред-


нему арифметическому первого и n-го членов этой прогрессии, умножив
на их количество:

Сумму первых членов арифметической прогрессии можно найти


по формуле:
( )

Геометрической прогрессией называют последовательность


, каждый член которой, начиная со второго, равен предыду-
щему и умножен на одно и то же число ( | | ), которое называ-
ют знаменателем геометрической прогрессии.
Пример: – геометрическая прогрессия, в которой

где | | (если же , то все члены прогрессии


равны первому члену).
Определяется n-й член геометрической прогрессии по формуле:

где – номер члена, – -й член, – первый член, – знамена-


тель прогрессии.
Модуль каждого члена геометрической прогрессии, начиная со
второго, является средним геометрическим двух соседних членов:
| | √
Сумму первых членов геометрической прогрессии можно найти
по формуле:

30
Если | | , то сумма бесконечной убывающей геометрической
прогрессии вычисляется по формуле:

Пример задачи с бесконечно убывающей прогрессией. Обра-


тить периодическую дробь ( ) в обыкновенную дробь.
Решение. Представим исходную дробь в виде бесконечной суммы:
( )

т.к. знаменатель бесконечно убывающей прогрессии .


Ответ: .

Однако это не единственный способ получить обыкновенную


дробь из бесконечной периодической дроби. Рассмотрим несколько при-
меров, в которых обратим внимание на запись чисел в скобках (период):

1) ( ) .
Как можно заметить, числитель в данном случае будет совпадать с
числом в скобках – это число «3», а знаменатель будет равен «9», т.к. в
скобках записана только одна цифра (если цифр будет две, то знаменатель
будет «99», если три – «999» и т.д.).
2) ( ) ( ) .

3) ( ) ( ) .
В данном случае для записи числителя необходимо из числа, обра-
зованного числами, стоящими после запятой – «342» отнять число, стоя-
щее до скобок – «34» – это и будет новый числитель нашей искомой дро-
би. Т.к. в скобках стоит только одна цифра «2» (период), то в знаменателе
будет одна «9», но благодаря тому, что до скобок стоит ещё два числа («3»
и «4»), то к «9» добавятся два нуля и получится конечный знаменатель –
«900», а далее просто сокращаем результат и получаем ответ.
4) ( ) ( ) .

31
ТЕМА 15. СИНУС, КОСИНУС, ТАНГЕНС И КОТАНГЕНС. РА-
ДИАННАЯ МЕРА
Угол – геометрическая фигура, образованная двумя лучами, вы-
ходящими из одной точки.
Кроме градусной меры, существует радианная мера измерения уг-
лов. Единицей радианной меры является радиан. Градусная мера развёр-
нутого угла равна , . Угол величиной 1 радиан – это угол с
вершиной в центре окружности, который опирается на дугу окружности и
длина этой окружности равна радиусу этого круга.
1 радиан = = 57о17′45″; рад.
Величина
угла в 0о 15о 30о 45о 60о 75о 90о 180о 360о
градусах
Величина
угла в
радианах
Переход от радианной меры угла к градусной и наоборот
Для того, чтобы перейти от градусной меры к радианной, нужно
разделить (т.е. ) на нужное число.
Для того, чтобы перейти от радианной меры к градусной, нужно
также разделить π (т.е. ) на знаменатель дроби.
Примеры:

Определения синуса, косинуса, тангенса и котангенса


Синус ( ) острого угла в прямоугольном треугольнике – это
отношение противолежащего катета к гипотенузе.
Косинус ( ) острого угла в прямоугольном треугольнике – это
отношение прилежащего катета к гипотенузе.
Тангенс ( ) острого угла в прямоугольном треугольнике – это
отношение противолежащего катета к прилежащему (отношение синуса
угла к его косинусу).
Котангенс ( ) острого угла в прямоугольном треугольнике –
это отношение прилежащего катета к противолежащему (отношение коси-
нуса угла к его синусу).

32
*Секанс ( ) острого угла в прямоугольном треугольнике – это
отношение гипотенузы к прилежащему катету; .
*Косеканс ( ) острого угла в прямоугольном треугольнике
– это отношение гипотенузы к противолежащему катету; .
При увеличении острого угла синус и тангенс угла возрастают, а
косинус и котангенс – убывают.

Тригонометрическая окружность

Знаки тригонометрических функций по четвертям

33
Таблица значений тригонометрических функций
15 18
0

√ √ √ √
1

√ √ √ √
0

√ √
√ √

√ √
√ 0 √

18 33 36

√ √ √ √

√ √ √ √
0

√ √
1 √ √

√ √
√ 1 0 √

Если к острому углу прибавить ( ) (т.е. 90о, 270о,


о
450 и т.д.), то первоначальная функция МЕНЯЕТСЯ на кофункцию.
Если к острому углу прибавить (т.е. 180о, 360о, 540о и
т.д.), то первоначальная функция НЕ МЕНЯЕТСЯ на кофункцию.
Формулы приведения тригонометрических функций

34
ТЕМА 16. ОСНОВНЫЕ ТРИГОНОМЕТРИЧЕСКИЕ ФОРМУЛЫ
Соотношения между тригонометрическими функциями одного и
того же аргумента:

Формулы сложения
( )
( )

( ) ( )

Формулы превращения суммы в произведение

(или );
( )

Формулы понижения степени


( )

Формулы половинного угла

35
Формулы двойного угла

Формулы произведения

( ( ) ( ))

( ( ) ( ))

( ( ) ( ))

Универсальная тригонометрическая подстановка

ТЕМА 17. ТРИГОНОМЕТРИЧЕСКИЕ И ОБРАТНЫЕ ФУНКЦИИ


Тригонометрические функции представляют собой элементар-
ные функции, аргументом которых является угол.
С помощью тригонометрических функций описываются соотно-
шения между сторонами и острыми углами в прямоугольном треугольнике
(вспомни тему 15, с.31).
Области применения тригонометрических функций чрезвычайно
разнообразны. Так, например, любые периодические процессы можно
представить в виде суммы тригонометрических функций (ряда Фурье).
Тригонометрические функции: синус, косинус, тангенс, котангенс,
секанс и косеканс.
Геометрическое определение тригонометрических функций
удобно ввести с помощью единичного круга (тригонометрической окруж-
ности). На рисунке ниже изображен круг единичного радиуса ( ). На
окружности обозначена точка ( ). Угол между радиус-вектором и
положительным направлением оси равен .

36
Синусом угла называется отношение ординаты точки ( )к
радиусу : .
Поскольку , то синус равен ординате точки ( ).
Косинусом угла называется отношение абсциссы точки ( )
к радиусу
Тангенсом угла называется отношение ординаты точки ( )к
eё абсциссе : .
Котангенсом угла называется отношение абсциссы точки
( ) к её ординате : .
Секанс угла − это отношение радиуса к абсциссе точки
( ): .
Косеканс угла − это отношение радиуса к ординате точки
( ): .

Тригонометрические функции и их графики


1) .
Область определения: .
Область значений: .
2) .
Область определения: .
Область значений: .

37
3) .
Область определения: ( ) .
Область значений: .
4) .
Область определения: .
Область значений: .
5) .
Область определения: ( ) .
Область значений: ( ).
6) .
Область определения: .
Область значений: ( ).

38
Обратные тригонометрические функции:
Арксинусом числа называется угол (число) из промежутка
[ ] синус которого равен .
Арккосинусом числа называется угол (число) из промежутка
косинус которого равен .
Арктангенсом числа наз. угол (число) из промежутка ( )
тангенс которого равен .
Арккотангенсом числа называется угол (число) из промежутка
( ) котангенс которого = .

ТЕМА 18. ТРИГОНОМЕТРИЧЕСКИЕ УРАВНЕНИЯ И НЕРА-


ВЕНСТВА

Уравнение Решение Частные случаи

)
| | , корней нет.
| | )
( )
или )

)
| | , корней нет.
| |
)

или )

39
Уравнения вида называ-
ют простейшими тригонометрическими уравнениями.
Чтобы рассуждения по нахождению корней этих уравнений были
более наглядными, воспользуемся графиками соответствующих функций и
разберём решение этих уравнений.

40
41
42
43
44
Как правило, решение сложных тригонометрических уравнений
сводится к решению простейших уравнений с помощью преобразований
тригонометрических выражений, разложения на множители и замены пе-
ременных. Если в уравнение, неравенство или тождество переменная вхо-
дит в одном и том же виде, то удобно соответствующее выражение с пере-
менной обозначить одной буквой, новой переменной (т.е. произвести за-
мену).

45
При поиске плана решения более сложных тригонометриче-
ских уравнений можно воспользоваться таким ориентиром:
 Пробуем привести все тригонометрические функции к одному ар-
гументу.
 Если удалось привести к одному аргументу, то пробуем все три-
гонометрические выражения привести к одной функции.
 Если к одному аргументу удалось привести, а к одной функции –
нет, тогда пробуем привести уравнение к однородному.
 В других случаях переносим все члены в одну сторону и пробуем
получить произведение ил используем специальные приемы ре-
шения.
Решение тригонометрических уравнений
приведением к одной функции

46
47
48
49
ТЕМА 19. УРАВНЕНИЯ И НЕРАВЕНСТВА С МОДУЛЕМ
Модулем числа называют расстояние (в единичных отрезках) от
начала координат до точки . Модуль числа равен . Модуль числа не
может быть отрицательным.
| | { | |

Модуль в уравнениях и неравенствах


( ) ( )
1. | ( )| | ( )| [
( ) ( )

( )
2. | ( )| ( ) { ( ) ( )
[
( ) ( )

( ) ( )
3. | ( )| ( ) [
( ) ( )

4. | ( )| | ( )| ( ( ) ( ))( ( ) ( ))

( ) ( )
5. | ( )| ( ) [
( ) ( )

( )
6. | ( )| ( ) { ( ) ( )
( ) ( )

( )
7. | ( )| ( ) { ( ) ( )
( ) ( )

ТЕМА 20. КОРЕНЬ n-й СТЕПЕНИ. СТЕПЕНЬ ЧИСЛА С РАЦИ-


ОНАЛЬНЫМ ПОКАЗАТЕЛЕМ

√ – корень степени; – показатель степени; – подкоренное


выражение.

50
Корнем n-й степени числа называется такое число, n-ая степень
которого равна числу .
Пример: √
Арифметическим корнем n-й степени из неотрицательного чис-
ла называется такое неотрицательное число, n-ая степень которого равна
числу .
Примеры: √ √
Корень чётной и корень нечётной степени
Корень чётной степени из отрицательного числа не определён:

Корень нечётной степени определен из любого числа:

Свойства корней n-й степени
1. √ √ √ 4. √ √
2. √
√ 5. √ (√ )

6. √ √
3. √ √ √ 7. √ √
Степенью числа с рациональным показателем , где
, ( ) называют число √


Степень числа определена только для положительных показа-
телей по определению:
для любого .

ТЕМА 21. СТЕПЕННАЯ ФУНКЦИЯ


Функция вида , где – независимая переменная (аргумент),
а – любое действительное число, называется степенной функцией.
На следующей странице представлены примеры некоторых сте-
пенных функций.

51
Некоторые графики степенных функций
1) 2)

3) 4)

На представленных выше и ниже графиках .

52
5) √ 6)

ТЕМА 22. ИРРАЦИОНАЛЬНЫЕ УРАВНЕНИЯ И НЕРАВЕН-


СТВА
Уравнения и неравенства, в которых под знаком корня имеется
переменная (неизвестная), называют иррациональными.
Решение иррациональных уравнений основывается на приведении
их с помощью некоторых превращений к рациональным уравнениям. Как
правило, это достигается возведением обеих частей уравнения в одну и ту
же степень (иногда несколько раз). При возведении обеих частей уравне-
ния в парную степень, полученное уравнение может иметь корни, которые
не удовлетворяют данному равенству (возникают посторонние корни).
Следует обязательно делать проверку полученных корней.
Пример: решите уравнение √ .
Решение: Т.к. в уравнении слева корень чётной степени, то
√ . Правая часть уравнения также должна
быть неотрицательной и тогда область значений
переменной Возведем в квадрат обе части: ( )
( ) , но
. Значит, уравнение имеет только один корень .
Ответ:

53
Условия равносильности в иррациональных уравнениях и неравенствах
( ) ( ( ))
1. √ ( ) ( ) {
( )
( )
2. I) √ ( ) √ ( ) { ( )
( ) ( )

√ ( ) ( ) ( ) ( )
II) √ ( ) √ ( ) { ( )
( )

III) √ ( ) √ ( ) ( ) (√ ( ) )

( ) ( ( )) ( ) ( ( ))
3. I. √ ( ) ( ) { ( ) II. √ ( ) ( ) { ( )
( ) ( )
( ) ( )
{ {
( ) ( )
4. I. √ ( ) ( ) [ ( ) II. √ ( ) ( ) [ ( )
{ {
( ) ( ( )) ( ) ( ( ))

( ) ( ) ( ) ( )
5. I. √ ( ) √ ( ) { II. √ ( ) √ ( ) {
( ) ( )
I)
√ ( ) ( ) ( ) ( )
а) √ ( ) √ ( ) { ( )
( )
√ ( ) ( ) ( ) ( )
б) √ ( ) √ ( ) { ( )
( )
II)
( ) (√ ( ) )
а) √ ( ) √ ( ) {
( )

б) √ ( ) √ ( ) ( ) (√ ( ) )
( ) ( ) ( ) ( )
6. I. √ ( ) √ ( ) { II. √ ( ) √ ( ) {
( ) ( )

54
ТЕМА 23. ПОКАЗАТЕЛЬНАЯ ФУНКЦИЯ И ПОКАЗАТЕЛЬНЫЕ
УРАВНЕНИЯ
Функцию вида , где называют показательной.
Показательная функция принимает только положительные значе-
ния, т.е. (всегда).
Основные свойства показательной функции
1. Область определений – множество действительных чисел ( ).
2. Область значений: ( ). Если , то .
3. Функция не является ни парной, ни непарной.
4. Если, , тогда функция возрастает; если , то
функция убывает.
5. При и , ; при , .
При и ; при .
6. График функции изображен на рисунке ниже.

Показательные уравнения
Показательными называют уравнения, в которых неизвестное
содержится в показателе степени при постоянных основаниях.
Примеры: – –

55
Простейшим показательным уравнением является уравнение
, где . Поскольку множество значений функции – мно-
жество положительных чисел, то уравнение :
1. Имеет один корень, если .
2. Не имеет корней, если .
Для решения показательных уравнений требуется знать и уметь
использовать следующую теорему:
( ) ( )
Показательное уравнение вида ( )
равносильно уравнению ( ) ( ).

ТЕМА 24. ЛОГАРИФМЫ. ЛОГАРИФМИЧЕСКАЯ ФУНКЦИЯ.


ЛОГАРИФМИЧЕСКИЕ УРАВНЕНИЯ И НЕРАВЕНСТВА
Логарифм положительного числа по основанию – это показа-
тель степени, в которую необходимо возвести , чтобы получить . Обо-
значение: . Читается: «логарифм числа по основанию ».
Определение логарифма можно кратко записать так:
Это равенство справедливо при и называется
основное логарифмическое тождество.
Свойства логарифмов
1.
2.
3.
4.
5. ( ) | | | |
6. | | | |
7. | |
8. | |

9. | || |
10.

11.

56
Логарифм числа
Уравнение , где , имеет единственный
корень. Его называют логарифмом числа по основанию и обозначают
.
Логарифмы по основанию 10 называют десятичными и обозна-
чаются ( ).
Логарифмы по основанию (экспонента, ) называют
натуральными и обозначают .
Логарифмическая функция
Функцию вида , называют логариф-
мической.
Основные свойства логарифмической функции

1. Область определения ( ) ( ).
2. Область значений ( ): множество действительных чисел .
3. Если , то .
4. Функция не является ни парной, ни непарной.
5. Если функция возрастает, а при убывает.
6. Если , то . Если и , то
.
7. Если и , то . Если и ,
то .

57
Логарифмические уравнения и неравенства
Логарифмическим уравнением называется уравнение, в котором
неизвестное содержится под знаком логарифма (в частности, в основании
логарифма).
Простейшее логарифмическое уравнение имеет вид: ( )
( ).
Решение любого логарифмического уравнения предполагает пере-
ход от логарифмов к выражениям, стоящим под знаком логарифмов. Од-
нако это действие расширяет область допустимых значений уравнения и
может привести к появлению посторонних корней. Чтобы избежать появ-
ления посторонних корней удобно использовать метод равносильного
перехода от исходного уравнения к системе, включающей ОДЗ:
( ) ( ) ( ) ( )
{ или {
( ) ( )
Если уравнение содержит неизвестное в основании логарифма:
( ) ( ) ( ) ( ), то переходим к системе:

( ) ( )
{ ( )
( ) ( )
Логарифмическим неравенством называется неравенство, в ко-
тором неизвестная величина стоит под знаком логарифма.
1) Неравенство ( ) : при сводится к ( ) , ес-
ли же , то ( ) .
2) Неравенство ( ) : при сводится к ( ) , ес-
ли же , то ( ) . Для получения решения нужно пересечь
неравенство с ОДЗ: ( ) (объединить в систему).

ТЕМА 25. ПРОИЗВОДНАЯ ФУНКЦИИ. ДИФФЕРЕНЦИРОВА-


НИЕ. НАХОЖДЕНИЕ ЭКСТРЕМУМА ФУНКЦИИ
Производная (функции в точке) – основное понятие дифференци-
ального исчисления, характеризующее скорость изменения функции (в
данной точке). Определяется как предел отношения приращения функции
к приращению её аргумента при стремлении приращения аргумента к ну-

58
лю, если такой предел существует. Функцию, имеющую конечную произ-
водную (в точке), называют дифференцируемой (в данной точке).
Определение производной функции через предел. Пусть в некото-
рой окрестности точки определена функция . Производной функ-
ции в точке называется предел, если он существует, ( )
( ) ( ) ( ) ( )
( )

Вспомним, что если функция ( ) определена в точках и


, то разность называют приращением аргумента (при переходе от
точки к точке ), а разность ( ) ( ) называют приращением
функции.
Приращение аргумента обозначают (читают: дельта икс; –
прописная буква греческого алфавита «дельта»; соответствующая строч-
ная буква пишется так: ). Приращение функции обозначают или .
Итак, , значит, .
( ) ( ) , значит, ( ) ( ).
Общепринятые обозначения производной функции ( ) в
точке : ( ) ( ) ( ) ( ) | ̇( )

Правила дифференцирования
Операция нахождения производной называется дифференциро-
ванием. При выполнении этой операции часто приходится работать с
частными, суммами, произведениями функций, а также с «функциями
функций», т.е. сложными функциями. Исходя из определения производ-

59
ной, можно вывести правила дифференцирования, облегчающие эту рабо-
ту. Если – постоянное число и ( ) ( ) – некоторые диффе-
ренцируемые функции, то справедливы правила дифференцирования:
  ( )
  ( )
 ( )
 ( )  ( )
Применение производных в физике
 ( ) ( )
 ( ) ( )

Геометрический смысл производной


Производная от функции в данной точке равна угловому
коэффициенту касательной, проведённой к графику функции в этой точке.

60
Рассмотрим две функции и нахождение их производных:
1. Функция ( ) . Производная ( ) .
Доказательство. Фиксируем ( ), придадим приращение ар-
гументу . Вычислим приращение функции:
( ) (( ) ), таким образом:
(( ) )
( )
2. Функция ( ) . Производная ( ) .
Доказательство. Фиксируем ( ), придадим приращение ар-
гументу . Вычислим приращение функции:
( ), таким образом:
( )
( )

Таблица производных функций ( )

Если данная в задании функция не (сложная), а (простая), то


и последний множитель просто исчезает (т.к. он равен ).
Если и – сложные функции, то ( ) вычисляется по формуле:
( ) ( )

61
Нахождение экстремума (точки минимума и максимума) функции
1. Находим производную функции.
2. Приравниваем эту производную к нулю.
3. Находим значения переменной получившегося выражения (зна-
чения переменной, при которых производная преобразуется в ноль).
4. Разбиваем этими значениями координатную прямую на проме-
жутки (при этом не нужно забывать о точках разрыва, которые также надо
наносить на прямую), все эти точки называются точками «подозрительны-
ми» на экстремум.
5. Вычисляем, на каких из этих промежутков производная будет
положительной, а на каких – отрицательной. Для этого нужно подставить
значение из промежутка в производную.
Из точек, подозрительных на экстремум, нужно найти именно экс-
тремумы. Для этого смотрим на наши промежутки на координатной пря-
мой. Если при прохождении через какую-то точку знак производной меня-
ется с плюса на минус, то эта точка будет максимумом, а если с минуса на
плюс, то минимумом.
Чтобы найти наибольшее и наименьшее значение функции, нужно
вычислить значение функции на концах отрезка и в точках экстремума.
Затем выбрать наибольшее и наименьшее значение и записать ответ.

ТЕМА 26. ПЕРВООБРАЗНАЯ. НЕОПРЕДЕЛЁННЫЙ И ОПРЕ-


ДЕЛЁННЫЙ ИНТЕГРАЛЫ
Функцию ( ) называют первообразной для функции ( ) на за-
данном промежутке, если для всех из этого промежутка ( ) ( ).
Функция ( ) для функции ( ) является первообраз-
ной, т.к. ( ) ( ) ( ).
Основное свойство первообразной
Если ( ) – первообразная для функции ( ) на заданном проме-
жутке, то ( ) имеет множество первообразных, и все эти первообразные
можно записать в виде ( ) , где – произвольная постоянная.
Функция ( ) является первообразной для функции
( ) , т.к. ( ) ( ) ( )

62
Правила вычисления первообразных
1. Первообразная суммы функций равна сумме первообразных
функций: т.е. если ( ) – первообразная для ( ), а ( ) – первообразная
для функции ( ) ( ).
2. Постоянный множитель можно выносить за знак первообраз-
ной, т.е. если ( ) – первообразная для функции ( )и – постоянная, то
( ) – первообразная для ( ).
3. Если ( ) – первообразная для ( ) и , – постоянная, то
( ) – первообразная для функции ( ).

Неопределённый интеграл
Неопределённым интегралом от функции ( ) называют выра-
жение ( ) , т.е. совокупность всех первообразных данной функции
( ). Обозначается так:
∫ ( ) ( ) ( )
где функцию ( ) называют подынтегральной функцией, выраже-
ние – подынтегральное выражение, ( ) – одна из первообразных
функции ( ), – произвольная постоянная.
Основные правила интегрирования
1. ∫( ( ) ( )) ∫ ( ) ∫ ( ) .
2. ∫ ( ) ∫ ( ) .
3. Если и – постоянные, то:

∫ ( ) ( )

Формулы вычисления неопределённых интегралов


1. ∫ ;
2. ∫ ∫ ;
3. ∫ ;
4. ∫ ;
5. ∫ ;

63
6. ∫ ∫ | | ;
7. ∫ √ ;

8. ∫ ;
9. ∫ ;
10. ∫ ;
11. ∫ | | ;
12. ∫ | | ;
13. ∫ ∫ ;
14. ∫ ∫ ;
15. ∫ | | ;
16. ∫ ;

17. ∫ ;
18. ∫ | √ | ;

19. ∫ | | ;
20. ∫ | | ;
21. ∫ ;
22. ∫ ;

23. ∫ ;
24. ∫ ;
25. ∫ ;
26. ∫ ;
( ) ( )
27. ∫ ∫ | ( )| .
( ) ( )

Пусть задана непрерырвная функция ( ), определённая на


промежутке , тогда определённым интегралом от числа до
функции ( ) называют приращение первообразной ( ) этой функции,

64
т.е. ∫ ( ) ( ) ( ) Числа и называют соответственно ниж-
ним и верхним пределами интегрирования.
Свойства определенных интегралов
1. ∫ ( ( ) ( )) ∫ ( ) ∫ ( )
2. ∫ ( ) ∫ ( )
3. ∫ ( ) ∫ ( )
4. ∫ ( ) ∫ ( ) ∫ ( )
5. ( ) ( ) ∫ ( ) ∫ ( )

Применение определённого интеграла


Площадь криволинейной трапеции (фигура, ограниченная гра-
фиком непрерывной положительной на промежутке функцией ( ),
осью и прямыми ) вычисляется по формуле:

∫ ( )

Площадь фигуры
Если на заданном промежутке непрерырвные функции
( ) и ( ) имеют свойственность ( ) ( ) для всех
, то:

∫( ( ) ( ))

65
ТЕМА 27. НАЧАЛА ТЕОРИИ ВЕРОЯТНОСТИ
Теория вероятности – это раздел математики, изучающий зако-
номерности массовых случайных явлений: случайные события, величины,
их свойства и операции над ними.
Событие – это явление, о котором можно сказать, что оно проис-
ходит или не происходит, в зависимости от природы самого события (обо-
значение: ). Любое событие происходит вследствие испытания
(подбрасывание монеты – испытание, появление герба – событие).
Событие называется случайным, если в результате опыта оно
может произойти или не произойти.
Событие называется достоверным, если оно обязательно про-
изойдет в результате данного опыта ( ), и невозможным, если оно не мо-
жет произойти в этом опыте ( ).
События называются равновозможными, если есть основания
считать, что ни одно из них не является более возможным, чем другое.
Два события называют совместными, если появление одного из
них не исключает появление другого в одном и том же испытании (два
стрелка стреляют по мишени, два спортсмена одновременно бегут и т.д.).
События называются несовместными, если появление одного из
них исключает появления других событий в одном и том же испытании
(день и ночь, студент едет на занятие и сдаёт экзамен) [логическая частица
«или»].
Теорема сложения вероятностей
Вероятность суммы двух несовместных событий равна сумме ве-
роятностей этих событий ( ( ) ( ) ( )), а совместных собы-
тий – сумме вероятностей этих событий без вероятности совместности
этих событий ( ( ) ( ) ( ) ( )).
Событие называется независимым, если вероятность появления
этого события не зависит от того произошло другое событие или нет (два
студента сдают экзамен – совместное и независимое), а зависимым – если
вероятность появления события зависит от того произошло или не про-
изошло событие (рабочий получил заработную плату в зависимости от
качества исполнения) [логическая частица «и»].

66
Теорема умножения вероятностей
Вероятность произведения двух независимых событий равна про-
изведению вероятностей этих событий ( ) ( ) ( ), а вероят-
ность произведения двух зависимых событий – произведению вероятности
одного из них на условную вероятность другого ( ) ( ) ( ).
Относительной частотой события называют отношение числа
испытаний, в которых событие появилось, к общему числу фактически
произведенных испытаний ( – число появлений события , – общее
число испытаний):
( )

Иногда относительную частоту события называют эмпирической


вероятностью. Относительная частота события носит случайный харак-
тер и будет меняться от серии к серии испытаний. При увеличении длины
серии испытаний эти колебания частоты от серии к серии будут стано-
виться меньше, поскольку случайные обстоятельства, влияющие на испы-
тания, в массе взаимно гасятся. Многочисленными экспериментами уста-
новлено, что:
 частоты ( ) с увеличением и проявляют тенденцию к стабилизации.
Экспериментальный факт стабилизации частоты получил название
статистической устойчивости частот;
 относительные частоты события при неограниченном увеличении
длины серии испытаний приближаются к вероятности события ( )
Конечно, при любом конечном числе опытов, вероятность собы-
тия в общем случае мы не получим, но приближенное значение вероятно-
сти появления события будем иметь.
Вероятность события равна отношению числа благоприятных
событию исходов опыта к общему числу исходов опыта.
( )
( )

где – событие, ( ) – вероятность события, – общее число


равновозможных и несовместимых событий; ( ) – число событий, кото-
рые содействуют событию (благоприятные).

67
ТЕМА 28. ЭЛЕМЕНТЫ КОМБИНАТОРИКИ
Множество – совокупность предметов, объектов или элементов.

Пустое множество – множество, не содержащее ни одного эле-


мента.
Если каждый элемент множества является также элементом
множества , то множество называется подмножеством множества
( и или, при ).
Перестановки (без повторений)
Перестановки – это различные комбинации, которые состоят из
элементов и отличаются порядком расположения этих элементов (порядок
выбора элементов важен).
Число перестановок из элементов (обозначается ) равно про-
изведению натуральных чисел от до , то есть («эн факториал»):

По определению .
Пример. На столе лежат фрукты: яблоко, груша и банан. Скольки-
ми способами мы можем их переставить между собой (порядок важен, т.е.
какой фрукт будет лежать первым, какой – вторым и т.д.)?
Ответ: нужно вычислить , т.е. существует 6
различных способов расположить данные фрукты между собой (я-г-б, я-б-
г, г-я-б, г-б-я, б-я-г, б-г-я).
Размещения (без повторений)
Число размещений – число комбинаций из элементов по эле-
ментов в каждой комбинации, которые отличаются либо составом элемен-
тов, либо порядком их расположения (порядок выбора элементов важен).
Число размещений из элементов по обозначают и оно рав-
няется произведению последовательных чисел, наибольшим из кото-
рых является :

( )
Если , то .

68
Примеры. 1) На столе лежат фрукты: яблоко, груша и банан.
Сколькими способами можно раздать по одному фрукту Даше и Кате?
Ответ: в данном случае сработает формула количества размеще-
ний, , мы учитываем все возможные перестановки объектов
( )
в каждой возможной выборке (выбираем 2 фрукта из 3-ёх и распределяем
между двумя девочками).
2) В классе учится 10 человек, нужно выбрать старосту группы,
заместителя старосты и ответственного за продовольствие. Сколько все-
возможных вариантов выбора 3-ёх таких людей из 10 возможных, если
порядок имеет значение (т.к. сначала выбираем старосту, потом замести-
теля старосты и затем – ответственного за продовольствие).
Ответ: в размещении порядок имеет значение! Вычислим:

( )
вариантов выбрать 3-ёх различных людей из
10 доступных.
Сочетания (без повторений)
Число сочетаний равно числу комбинаций из элементов по
элементов в каждой комбинации, которые отличаются только составом
элементов (порядок выбора элементов не важен).
Число сочетаний из элементов по ( ) обозначают
символом и вычисляют по формуле:

( )
Пример. В классе учится 10 человек, нужно выбрать 3-ёх дежур-
ных. Сколькими способами можно выбрать 3-ёх дежурных?
Ответ: здесь уже порядок выбора не важен (дежурные равнознач-
ны), поэтому применяем формулу сочетаний: ( )
.
Если вернуться к примеру с фруктами, то число покажет
сколькими способами можно выбрать 1 фрукт из 3-ёх доступных (порядок
опять таки – не важен).

69
ГЕОМЕТРИЯ
Геометрия – это наука, занимающаяся изучением геометрических
фигур и их свойств.
Школьный курс геометрии включает в себя планиметрию и сте-
реометрию. Общепринятую в наши дни классификацию различных разде-
лов геометрии предложил Феликс Клейн в своей «Эрлангенской програм-
ме» (1872). Согласно Клейну, каждый раздел изучает те свойства геомет-
рических объектов, которые сохраняются (инвариантны) при действии
некоторой группы преобразований, специфичной для каждого раздела. В
соответствии с этой классификацией, в классической геометрии можно
выделить следующие основные разделы.
Евклидова геометрия, в которой предполагается, что размеры
отрезков и углов при перемещении фигур на плоскости не меняются. Дру-
гими словами, это теория тех свойств фигур, которые сохраняются при их
переносе, вращении и отражении.
Планиметрия – раздел евклидовой геометрии, исследующий фи-
гуры на плоскости.
Стереометрия – раздел евклидовой геометрии, в котором изуча-
ются фигуры в пространстве.
Проективная геометрия, изучает проективные свойства фигур,
то есть свойства, сохраняющиеся при их проективных преобразованиях.
Инварианты в этой геометрии – это свойства, сохраняющиеся при замене
фигур на подобные им, но другого размера.
Аффинная геометрия, использующая очень общие аффинные
преобразования. В ней длины и величины углов не имеют существенного
значения, но прямые переходят в прямые.
Начертательная геометрия – инженерная дисциплина, в основе
которой лежит метод проекций. Этот метод использует две и более проек-
ций (ортогональных или косоугольных), что позволяет представить трёх-
мерный объект на плоскости.
Аналитическая геометрия – геометрия координатного метода. В
ней геометрические объекты описываются алгебраическими уравнениями
в декартовых (иногда аффинных) координатах и затем исследуются мето-
дами алгебры и анализа.

70
Дифференциальная геометрия – изучает линии и поверхности,
задающиеся дифференцируемыми функциями, с помощью дифференци-
альных уравнений.
Геометрия Лобачевского (или гиперболическая геометрия) – од-
на из неевклидовых геометрий, геометрическая теория, основанная на тех
же основных аксиомах, что и обычная евклидова геометрия, за исключе-
нием аксиомы о параллельных прямых, которая заменяется её отрицанием.

ТЕМА 1. ПРОСТЕЙШИЕ ГЕОМЕТРИЧЕСКИЕ ФИГУРЫ И ИХ


СВОЙСТВА
Начинаем изучение геометрии с раздела планиметрия. Простей-
шими фигурами на плоскости являются точка и прямая.
Точка – самая простая геометрическая фигура. Это единственная
фигура, которую нельзя разбить на части.
Определение прямой начинается с определения линии. Линия –
это множество точек, соединённых между собой. Линия может быть пря-
мой, кривой, ломанной, непрерывной и даже разомкнутой. И именно из-за
этого разнообразия линии очень трудно определить в пространстве. Непо-
нятно, как пройдет та или иная кривая, когда выйдет за пределы листа.
Поэтому был выделен отдельный вид линий – прямые. Прямые – это бес-
конечные непрерывные линии, которые не имеют искривлений. Главное
правило линий – через любые две точки можно провести линию.
Прямая – это геометрическая фигура, обладающая определённы-
ми свойствами.
Основное свойство прямой: через любые две точки можно про-
вести прямую, и притом только одну.
Пересекающиеся прямые – две (или более) прямые, имеющие
общую точку.
Аксиомы планиметрии
Аксиомы принадлежности точек и прямых на плоскости:
1. Какой бы ни была прямая, существуют точки, ей принадлежа-
щие и ей не принадлежащие.
2. Через две точки можно провести прямую, и только одну.

71
Аксиомы взаимного размещения точек на прямой и на плоскости:
1. Из трех точек на прямой одна и только одна лежит между двумя
другими.
2. Прямая разбивает плоскость на две полуплоскости.
3. Отрезок MN пересекает прямую a, если точки M и N лежат в
разных полуплоскостях относительно прямой a.
Аксиомы измерения отрезков и углов:
1. Каждый отрезок имеет определенную длину, больше нуля. Дли-
на отрезка равна сумме длин частей, на которые разбивается отрезок лю-
бой своей точкой.
Расстояние между двумя точками – это длина отрезка, соединя-
ющего эти точки.
2. Каждый угол имеет определенную градусную меру, больше ну-
ля. Развернутый угол равен 180о. Градусная мера угла равна сумме градус-
ных мер углов, на которые он разбивается любым лучом, проходящим
между его сторонами.
Аксиомы отложения отрезков и углов:
1. На любой полупрямой от ее начальной точки можно отложить
отрезок заданной длины, и только один.
2. От любой полупрямой в заданную полуплоскость можно отло-
жить угол с заданной градусной мерой, меньше , и только один.
3. Каким бы ни был треугольник, существует равный ему тре-
угольник в заданном размещении относительно данной полупрямой.
Углы
Угол – геометрическая фигура, образованная двумя лучами, вы-
ходящими из одной точки.

Биссектрисой называется луч, проходящий между сторонами уг-


ла и делящий угол на две равные части.

72
Виды углов
о
прямой (90 ); тупой (больше 90о);
острый (меньше 90о); развернутый (180о).
Смежные и вертикальные углы
Смежными углами называются углы, у которых одна сторона
общая, а две другие стороны этих углов являются дополняющими полу-
прямыми. Сумма смежных углов (рис. а) равна ( ).

а) б)
Если два угла равны, то смежные с ними углы тоже равны. Угол,
смежный с прямым углом, является прямым.
Вертикальными называются углы, у которых стороны являются
дополняющими полупрямыми. Вертикальные углы (рис. б) равны (
и ).
Параллельные и перпендикулярные прямые
Две прямые на плоскости называются параллельными, если они
не пересекаются ( || , рис. в).
Две прямые называются перпендикулярными, если они пересе-
каются под прямым углом. Через данную точку можно провести прямую,
перпендикулярную данной, и только одну ( , рис. г).

в) г)

73
Аксиома параллельных прямых: через точку, не лежащую на
данной прямой, можно провести на плоскости прямую, параллельную дан-
ной, и только одну.
Углы при пересечении двух прямых секущей
При пересечении двух прямых третьей, секущей, образуются сле-
дующие виды углов:
а) внутренние односторонние: 1 и 8, 4 и 5;
б) внутренние разносторонние (накрест лежащие): 1 и 5, 4 и 8;
в) соответственные: 1 и 7, 4 и 6, 2 и 8, 3 и 5.

Признаки параллельности двух прямых


1. Если при пересечении двух прямых секущей накрест лежащие
углы равны, то прямые параллельны.
2. Если при пересечении двух прямых секущей соответственные
углы равны, то прямые параллельны.
3. Если при пересечении двух прямых секущей сумма внутренних
односторонних углов равна , то прямые параллельны.
4. Две прямые, перпендикулярные третьей прямой, параллельны.
5. Две прямые, параллельные третьей, параллельны друг другу.
Свойства углов, образованных при пересечении параллельных прямых
секущей
1. Если две параллельные прямые пересечены секущей, то накрест
лежащие углы равны.
Следствие. Если прямая перпендикулярна к одной из двух парал-
лельных прямых, то она перпендикулярна и к другой.

74
2. Если две параллельные прямые пересечены секущей, то соот-
ветственные углы равны.
3. Если две параллельные прямые пересечены секущей, то сумма
односторонних углов равна .
Признак перпендикулярности прямых
Если две пересекающиеся прямые соответственно параллельны
двум перпендикулярным прямым, то они перпендикулярны.
Признаки перпендикулярности прямой и плоскости
1) Если прямая перпендикулярна двум пересекающимся прямым,
лежащим в плоскости, то она перпендикулярна этой плоскости.
2) Если плоскость перпендикулярна одной из параллельных пря-
мых, то она перпендикулярна и другой.

ТЕМА 2. ТРЕУГОЛЬНИК. ПРИЗНАКИ РАВЕНСТВА ТРЕ-


УГОЛЬНИКОВ. ВИДЫ ТРЕУГОЛЬНИКОВ
Треугольник – это геометрическая фигура, образованная тремя
отрезками, которые соединяют три точки, не лежащие на одной прямой.
Указанные точки называются вершинами, а отрезки – сторонами тре-
угольника. Обозначение треугольника: .
Медиана – отрезок, соединяющий вершину треугольника с сере-
диной противолежащей стороны.
Свойства медианы
1. Все медианы треугольника пересекаются в одной точке, которая
называется центроидом (центром тяжести
треугольника), и делятся этой точкой на две
части в отношении 2:1, считая от вершины:

2. Любая медиана треугольника де-


лит его на два равновеликих треугольника ( ).
3. Весь треугольник разделяется своими медианами на шесть рав-
новеликих треугольников ( ).

75
Биссектрисой треугольника называется отрезок, который соеди-
няет вершину угла с противолежащей стороной и делит угол пополам.
Биссектрисы треугольника пересекаются в одной точке – центре
вписанной окружности, которая называется инцентром треугольника.
Основное свойство биссектрисы
Биссектриса делит противолежащую сторону
на части, пропорциональные к прилежащим сторонам:

Высотой треугольника называется


перпендикуляр, опущенный из его вершины к
прямой, содержащей противолежащую сто-
рону треугольника.
Высоты треугольника пересекаются в
одной точке, которая называется ортоцен-
тром треугольника.
Средняя линия треугольника – от-
резок, соединяющий середины двух сторон
треугольника (MN – средняя линия).
Свойства средней линии
1. Средняя линия треугольника парал-
лельна одной из его сторон и равна половине её длины.
2. Три средние линии треугольника делят его на четыре равных
треугольника (подобных данному, с коэффициентом подобия ).
Треугольники будут называться равными, если равны их соответ-
ственные элементы: стороны, углы, медианы, биссектрисы и высоты.
Признаки равенства треугольников
Первый признак равенства треугольников – по двум сторонам и
углу между ними.
Если две стороны и угол между ними одного треугольника соот-
ветственно равны двум сторонам и углу между ними другого треугольни-
ка, то такие треугольники равны.

76
Второй признак равенства треугольников – по стороне и приле-
жащим к ней углам.
Если сторона и два прилежащих к ней угла одного треугольника
равны соответственно стороне и двум прилежащим к ней углам другого
треугольника, то такие треугольники равны.
Третий признак равенства треугольников – по трём сторонам.
Если три стороны одного треугольника равны соответственно
трём сторонам другого треугольника, то такие треугольники равны.
Формула нахождения длины биссектрисы треугольника, проведённую к
стороне
√ ( )
,
где и – стороны -ка, а – угол между ними, .
Формулы нахождения длины медианы тре-
угольника
√ ( )

Виды треугольников
Треугольник называется равнобедренным,
если у него две стороны равны. Эти равные стороны
называются боковыми сторонами, а третья сторона
называется основанием треугольника.
Свойства равнобедренного треугольника:
1. У равнобедренного треугольника две
равные стороны (боковые).
2. У равнобедренного треугольника углы при основании равны.
3. У равнобедренного треугольника медиана, проведенная к осно-
ванию, является биссектрисой и высотой.
Признаки равнобедренного треугольника:
1. Треугольник является равнобедренным, если в нём совпадают:
а) высота и медиана;

77
б) высота и биссектриса;
в) медиана и биссектриса.
1. Если в треугольнике два угла равны, то он равнобедренный.
Треугольник, у которого все стороны рав-
ны, называется равносторонним или правиль-
ным.
Свойства равностороннего -ка:
1. У равностороннего треугольника все
углы равны (по ).
2. Любая медиана равностороннего тре-
угольника является биссектрисой и высотой.
Треугольник называется остроугольным, если все три его угла –
острые, то есть меньше .
Треугольник называется тупоугольным, если один из его углов –
тупой, то есть больше .
Треугольник называется прямоугольным,
если у него есть прямой угол, то есть угол в .
Сторона прямоугольного треугольника,
противолежащая прямому углу, называется гипо-
тенузой (AB), две другие стороны называются ка-
тетами (BC и AC).
Свойства прямоугольного треугольника:
1. В прямоугольном треугольнике против угла лежит ка-
тет, равный половине гипотенузы.
2. Медиана прямоугольного треугольника, проведенная к гипоте-
нузе, равна половине гипотенузы.
3. В прямоугольном треугольнике катет есть среднее геометриче-
ское гипотенузы и его проекции на эту гипотенузу:
√ √
4. В прямоугольном треугольнике высота (проведенная из прямого
угла) есть среднее геометрическое для отрезков, на которые делится гипо-
тенуза этой высотой (проекции катетов):

78
Проекция катета – это отрезок, заключенный между самим кате-
том и высотой, проведённой из вершины прямого угла.
Признаки равенства прямоугольных треугольников
1-ый признак: по двум катетам.
2-ой признак: по гипотенузе и катету.
3-ий признак: по гипотенузе и острому углу.
4-ый признак: по катету и острому углу.

ТЕМА 3. СУММА УГЛОВ ТРЕУГОЛЬНИКА. НЕРАВЕНСТВО


ТРЕУГОЛЬНИКА.
1. Сумма углов треугольника равна .
2. В любом треугольнике два угла – острые.
3. Если один из углов равнобедренного треугольника равен , то
этот треугольник – равносторонний (т.е. у равностороннего треугольника
все углы равны ).
Внешним углом треуголь-
ника при данной вершине называется
угол, смежный с углом треугольника
при этой вершине ( ).
Внешний угол треугольника
равен сумме двух внутренних углов,
не смежных с ним (т.к. сумма смеж-
ных углов равна и сумма углов
треугольника равна ).
Центр окружности, вписанной в равнобедренный треугольник,
принадлежит медиане, высоте и биссектрисе, проведенным с вершины к
основанию.
Центры окружности, описанной около равностороннего треуголь-
ника, и окружности, вписанной в него, совпадают. Это точка пересечения
медиан, биссектрис и высот равностороннего треугольника.
Неравенство треугольника: длина любой стороны треугольника
всегда меньше суммы длин двух других его других сторон.

79
Обратите внимание, что если большая сторона треугольника
меньше суммы двух других сторон, то остальные стороны можно не про-
верять по неравенству треугольника, оно уже нарушится. Значит, начинать
проверку на существование треугольника с заданными сторонами удобно с
большей стороны.

ТЕМА 4. ПОДОБИЕ ТРЕУГОЛЬНИКОВ. СВОЙСТВА ПОДОБ-


НЫХ ТРЕУГОЛЬНИКОВ. ТЕОРЕМА ПИФАГОРА
Признаки подобия треугольников
Первый признак подобия треугольников (по двум углам). Если
два угла одного треугольника соответственно равны двум углам другого
треугольника, то такие треугольники являются подобными.
Второй признак подобия треугольников (по двум сторонам и уг-
лу между ними). Если две стороны одного треугольника пропорциональ-
ны двум сторонам другого треугольника и углы, созданные этими сторо-
нами, равны, то такие треугольники являются подобными.
Третий признак подобия треугольников (по трём сторонам). Если
три стороны одного треугольника пропорциональны трем сторонам друго-
го треугольника, то такие треугольники являются подобными.
Коэффициентом подобия называют число , равное отношению
сходственных (соответственных, т.е. к которым прилегают одинаковые
углы) сторон подобных треугольников.
Свойства подобных треугольников
1) Отношение длин соответствующих элементов подобных тре-
угольников (в частности, длин биссектрис, медиан, высот и серединных
перпендикуляров) равно коэффициенту подобия.
2) Отношение периметров подобных треугольников равно коэф-
фициенту подобия: .
3) Отношение площадей двух подобных треугольников равно
квадрату коэффициента подобия:
4) Объемы (и массы) подобных тел (в пространстве) относятся
как куб коэффициента подобия

80
Теорема Пифагора
В прямоугольном треугольнике сумма
квадратов катетов равна квадрату гипотенузы.



Теорема, обратная теореме Пифагора. Если квадрат одной сто-
роны треугольника равен сумме квадратов двух других сторон, то такой
треугольник будет прямоугольным.

ТЕМА 5. ПЛОЩАДЬ ТРЕУГОЛЬНИКА. ПОНЯТИЕ ПЛОЩАДИ


Площадь (S) – это численная характеристика двумерной геомет-
рической фигуры, показывающая размер этой фигуры. Исторически вы-
числение площади называлось квадратурой. Фигура, имеющая площадь,
называется квадрируемой. Рассмотрим формулы нахождения площади
треугольников:

Формулы нахождения высоты и стороны равностороннего тре-


√ √
угольника: .

81
ТЕМА 6. ОКРУЖНОСТЬ И КРУГ. КАСАТЕЛЬНАЯ К ОКРУЖ-
НОСТИ
Геометрическим местом точек называют фигуру, которая состоит
из всех точек плоскости, которые имеют определенные свойства.
Окружность – это геометрическое место точек плоскости, равно-
удаленных от данной точки, которая называется центром.
Часть плоскости (конечная), ограниченная окружностью называ-
ется кругом. Если радиус равен нулю, то круг вырождается в точку.
Радиус ( ) – это расстояние от центра
окружности до произвольной точки на окружности (OA,
OB, OC).
Хорда – это отрезок, который соединяет две
точки окружности (BC).
Диаметр ( ) – хорда, проходящая через
центр окружности (AC).

Свойства хорд
1. Равные хорды окружности равноудалены от центра и имеют
одинаковую длину.
2. Если две хорды окружности пересекаются, то произведение
отрезков одной хорды равно произведению отрезков другой хорды.
3. Дуги, заключенные между параллельными хордами, равны.
4. Диаметр (радиус), перпендикулярный к хорде, делит эту хорду и
обе стягиваемые ею дуги пополам. Верно и обратное утверждение: если
диаметр (радиус) делит пополам хорду, то он перпендикулярен этой хорде.
Прямая, имеющая только одну общую точку с
окружностью, называется касательной к окружности,
а их общая точка называется точкой касания прямой
и окружности. На рисунке слева – точка О – центр
окружности, прямая АВ – касательная.
Свойства касательной
1. Касательная к окружности перпендикулярна радиусу, проведен-
ному в точку касания (OM AB).

82
2. Отрезки касательных к окружности,
проведенных из одной точки, равны и составляют
равные углы с прямой, проходящей через эту точ-
ку и центр окружности.
Теорема о касательной и секущей. Если из
точки, лежащей вне окружности, проведены касатель-
ная и секущая, то квадрат длины касательной равен
произведению секущей на её внешнюю часть:

Свойство касательной и секущей, пересе-


кающихся вне окружности. Если касательная и секущая пересекаются в
одной точке (M), то угол между касательной и секущей будет равен поло-
вине разности большей и меньшей градусных мер отсекаемых дуг, т.е.
( ) .
Также данная формула применима и в случае
пересечения двух секущих вне окружности, в т. M:
( ) .
Теорема о секущих. Если из точки, лежащей
вне окружности, проведены две секущие, то произведе-
ние одной секущей на её внешнюю часть равно произ-
ведению другой секущей на её внешнюю часть: .

ТЕМА 7. ЦЕНТРАЛЬНЫЙ И ВПИСАННЫЙ УГЛЫ


Центральный угол — это угол, вершина ко-
торого находится в центре окружности.
Угол АОВ – центральный (рисунок а).
Вписанным называется угол, вершина кото-
рого лежит на окружности, а стороны пересекают её.
Угол АВС – вписанный (рисунок б).
Часть окружности, ограниченная двумя точ-
ками, называется дугой окружности, градусная мера
которой соответствует данному центральному углу.
Дуги на рисунках выделены: АВ и AC.

83
Градусной мерой дуги окружности называется градусная мера
соответственного центрального угла.
Градусная мера окружности равна .
Свойства вписанных углов
1. Вписанные углы, опирающиеся на одну и ту же дугу, равны.
2. Вписанный угол, опирающийся на диаметр, – прямой.
3. Величина вписанного угла в два раза меньше центрального,
опирающегося на ту же дугу.

ТЕМА 8. ДЛИНА КРУГА И ПЛОЩАДЬ КРУГА


Длина окружности (круга) вычисляется
по формуле:

где – радиус окружности (круга), –


его диаметр.
Длину дуги окружности (круга) радиуса
можно вычислить по формуле ( – градусная мера
дуги, – радиус):

Площадь круга вычисляется по формуле:

где – радиус круга, – диаметр, –


число Архимеда ( ).
Круговым сектором называется часть
круга, которая ограничена дугой окружности и
двумя радиусами (рис. а).
Площадь сектора вычисляется по форму-
ле:

где – радиус, – градусная мера дуги.

84
Круговым сегментом называется часть круга, ограниченная дугой
окружности и стягивающей её хордой (рис. б).
Если круг пересечь прямой, он делится на два круговых сегмента.
Площадь кругового сегмента вычисляется по формулам:

Круговое кольцо – это часть плоскости, ограниченная двумя


окружностями с разными радиусами (рис. в).
Площадь кругового кольца равна разнице площадей внешнего и
внутреннего кругов.
( ) ( )
где ( ) – диаметр (радиус) внешнего круга, а ( ) – диаметр (ра-
диус) внутреннего круга.

ТЕМА 9. ВПИСАННЫЕ И ОПИСАННЫЕ ОКРУЖНОСТИ


Прямая, которая перпендикулярна к одной из
сторон треугольника и делит её пополам, называется
серединным перпендикуляром. На рисунке слева
серединные перпендикуляры пересекаются в точке О.
Окружность называется описанной вокруг
треугольника, если она проходит через все его верши-
ны. Треугольник при этом является вписанным.
Вокруг любого -ка можно описать только
одну окружность. Центром окружности, описанного
вокруг -ка, является точка пересечения серединных
перпендикуляров к сторонам -ка.
Формула для – радиуса описанной окружно-
сти, в которой: – длины сторон -ка, A, B, C – вершины (углы) -ка,
– полупериметр -ка, – площадь -ка.

√ ( )( )( )

85
Радиус окружности описанной вокруг равностороннего треуголь-
ника со стороной можно вычислить по формуле:


Радиус окружности, описанной вокруг прямоугольного треуголь-
ника с катетами и гипотенузой , можно вычислить по формуле:

Окружность называется вписанной в


треугольник, если она касается всех его сторон.
Треугольник при этом называется описанным
вокруг окружности.
В любой треугольник можно вписать
только одну окружность. Центром окружно-
сти, вписанной в треугольник, является точкой
пересечения его биссектрис. Для четырехугольника ситуация та же.
Радиус вписанной в треугольник окружности находим так:
( )( )( )

где – длины сторон треугольника, – площадь треугольни-


ка, – полупериметр.
Радиус окружности, вписанной в равносторонний -к со сторо-
ной , можно вычислить по формуле ( – радиус описанной окружности):


Радиус окружности, вписанной в прямоугольный треугольник с
катетами и гипотенузой , можно вычислить по формуле:

ТЕМА 10. ТЕОРЕМА КОСИНУСОВ И СИНУСОВ


Для нахождения элементов в произвольном треугольнике исполь-
зуется теорема синусов или теорема косинусов, разберём их ниже.

86
Теорема косинусов
Квадрат любой стороны -ка равен сумме квадратов двух
других его сторон, без удвоенного произведения этих сторон на коси-
нус угла между ними.

Доказательство. Рассмотрим тре-


угольник . Из вершины на сторону
опущена высота . Из треугольника
следует:

Запишем теорему Пифагора для


двух прямоугольных треугольников и :
( ) ( )и ( ) ( )
Приравниваем правые части уравнений ( ) и ( ) и:
( ) ( ) или
Случай, когда один из углов при основании тупой (и высота пада-
ет на продолжение основания), полностью аналогичен рассмотренному.
Выражения для сторон и :

Теорема синусов
Стороны треугольника пропорциональны синусам противо-
лежащих углов.

где — стороны треугольника, — соответственно про-


тиволежащие им углы, а — радиус описанной около треугольника
окружности.
Доказательство. Достаточно
доказать равенство:
Проведем диаметр | | для
описанной окружности. По свойству
углов, вписанных в окружность,

87
прямой и угол при вершине треугольника равен либо , если
точки и лежат по одну сторону от прямой , либо в против-
ном случае. Поскольку ( ) , в обоих случаях .
Повторив тоже рассуждение для двух других сторон треугольника
получаем:

ТЕМА 11. ЧЕТЫРЁХУГОЛЬНИКИ: ПАРАЛЛЕЛОГРАММ,


ПРЯМОУГОЛЬНИК, РОМБ, КВАДРАТ, ТРАПЕЦИЯ
Четырёхугольником называется
фигура, состоящая из четырёх точек и четы-
рёх отрезков, которые последовательно их
соединяют. При этом ни одна из трёх данных
точек не должна лежать на одной прямой, а
отрезки, которые их соединяют, не должны
пересекаться. Данные точки называются вершинами четырёхугольника, а
отрезки, которые их соединяют, – сторонами четы-
рёхугольника. Вершины четырёхугольника называют-
ся соседними, если они являются концами одной из
его сторон. Не соседние вершины называются проти-
волежащими.
Отрезки, соединяющие противолежащие вер-
шины четырёхугольника, называются диагоналями.
Стороны четырёхугольника, исходящие из одной вершины, назы-
ваются соседними, а стороны, не имеющие общих вершин, называются
противолежащими.
Четырёхугольник называется выпуклым (рис.
а), если он лежит по одну сторону относительно прямой,
проходящей через любые две его вершины (каждый из его
углов меньше ). В противном случае четырёхугольник
называется невыпуклым (вогнутым, рис. б) (существует
хотя бы один угол от до ). Диагонали выпуклого
четырёхугольника лежат внутри него и пересекаются.

88
Сумма внутренних углов любого четырёхугольника равна .
Периметр – сумма длин всех сторон.
Параллелограмм
Параллелограмм – это четырехугольник, у которого противоле-
жащие стороны попарно параллельны и равны.
Свойства параллелограмма
1. Противолежащие стороны равны (то есть ).
2. Противолежащие углы равны
( ).
3. Диагонали точкой пересечения
делятся пополам.
4. Каждая диагональ разбивает па-
раллелограмм на два равных треугольника.
5. Сумма углов, прилежащих к одной стороне параллелограмма,
равна .
6. Сумма квадратов диагоналей параллелограмма равна сумме
квадратов его сторон.
Прямоугольник
Прямоугольником называется параллелограмм, у которого все
углы прямые.
Свойства прямоугольника
Прямоугольник имеет все свойства па-
раллелограмма, кроме того, диагонали прямо-
угольника равны между собой, а все углы –
прямые.
Ромб
Ромб – параллелограмм, у которого все
стороны равны.
Свойства ромба
Ромб имеет все свойства параллело-
грамма, а также его диагонали взаимно перпен-
дикулярны и являются биссектрисами углов.

89
Квадрат
Квадрат – прямоугольник, у которого все сто-
роны равны; или ромб, у которого все углы прямые.
Свойства квадрата
1. Квадрат имеет все свойства прямоугольника и
ромба.
2. У квадрата все углы прямые и все стороны равны.
3. Диагонали квадрата равны и пересекаются под прямым углом.
4. Диагонали квадрата являются биссектрисами его углов. Каждая
диагональ квадрата создает со стороной угол .
Трапеция
Трапеция – четырехугольник, у которого две стороны параллель-
ны, а две другие – нет.
Параллельные стороны трапеции назы-
ваются основаниями, а непараллельные сторо-
ны – боковыми. На рисунке слева ABCD – тра-
пеция, поскольку AD BC, а AB CD, стороны
BC и AD – основания трапеции, AB и CD – боко-
вые стороны трапеции.
Высотой трапеции называют перпендикуляр, проведенный из
любой точки одного из оснований на прямую, которая содержит второе
основание (или это расстояние между основаниями трапеции). На рисунке
BH – высота трапеции ABCD.
Средней линией трапеции называют отрезок, который соединяет
середины боковых сторон трапеции. На рисунке EF – средняя линия тра-
пеции ABCD.
Свойства трапеции
1. Сумма углов трапеции, прилежащих к боковой стороне, равна
.
2. Средняя линия трапеции параллельна основаниям и равна их
полусумме:
3. Сумма квадратов диагоналей трапеции равна сумме квадратов
боковых сторон полюс удвоенное произведение оснований этой трапеции.

90
в) г)
Трапеция, у которой боковые стороны равны, называется рав-
нобокой (равнобедренной) трапецией (рис. в).
Свойства равнобокой трапеции
1. Углы при основании равны.
2. Диагонали равны.
3. Если в равнобокой трапеции диагонали взаимно перпендику-
лярны, то её высота равна средней линии.
4. Окружность можно описать только вокруг равнобедренной тра-
пеции.
5. Если в равнобокую трапецию вписать окружность, то её боковая
сторона равна средней линии.
Прямоугольной называется трапеция, у которой одна из боковых
сторон перпендикулярна основаниям (рис. г). Эта сторона также является
высотой трапеции.

ТЕМА 12. ПЛОЩАДИ ЧЕТЫРЕХУГОЛЬНИКОВ


Площадь прямоугольника равна произведению двух его смежных
сторон:

Площадь прямоугольника равна половине квадрата его диагонали,


умноженной на синус угла между диагоналями:

Площадь квадрата равна квадрату его стороны:

Площадь квадрата равна половине квадрата его диагонали:

91
Площадь параллелограмма (и ромба) равна произведению его сто-
роны (основания) на высоту, проведённую к ней:

Площадь параллелограмма равна произведению двух его смежных


сторон на синус угла между ними:

Площадь параллелограмма равна половине произведения его диа-


гоналей на синус угла между ними (также эта формула верна и для про-
извольного четырёхугольника):

Площадь ромба равна произведению квадрата его стороны на си-


нус угла ромба:

Площадь ромба равна половине произведения его диагоналей:

Площадь трапеции равна произведению полусуммы оснований на


высоту:

Площадь трапеции равна произведению средней линии на высоту:

Если диагонали равнобедренной трапеции взаимно перпендику-


лярны, то: , где – высота трапеции, – средняя линия.
Формула Брахмагупты (только для вписанного в окружность че-
тырёхугольника, где – длины сторон, а – полупериметр):
√( )( )( )( )

ТЕМА 13. ЛОМАНАЯ. МНОГОУГОЛЬНИКИ


Ломаной называется фигура, состоящая из точек
и соединяющих их отрезков . Точки
называются вершинами ломаной, а отрезки

92
– звеньями ломаной. Ломаная называется простой,
если она не имеет самопересечений.

Ломаная называется замкнутой, если её концы совпадают.


Длиной ломаной называется сумма длин её звеньев. Длина лома-
ной не меньше длины отрезка, соединяющего её концы.
Выпуклые многоугольники
Простая замкнутая ломаная называется многоугольником, если
её соседние звенья не лежат на одной прямой. Вершины ломаной называ-
ется вершинами многоугольника, а звенья ломаной – сторонами много-
угольника. Отрезки, соединяющие несоседние вершины многоугольника,
называются диагоналями. Многоугольник с n-вершинами (и n-сторонами)
называется n-угольником.
Плоским многоугольником, или много-
угольной областью, называется конечная часть
плоскости, ограниченная многоугольником.
Многоугольник называется выпуклым, если
он лежит в одной полуплоскости относительно любой
прямой, содержащей его сторону. При этом сама
прямая считается принадлежащей полуплоскости.
Углом выпуклого многоугольника при данной вершине называет-
ся угол, образованный его сторонами, которые сходятся в этой вершине.
Внешним углом выпуклого многоугольника при данной вершине называ-
ется угол, смежный внутреннему углу многоугольника при этой вершине.
Сумма углов выпуклого n-угольника равна ( )
Сумма внешних углов выпуклого n-угольника, взятых по одному
при каждой вершине, равна
Каждый внутренний угол правильного n-угольника равен
( )
, а каждый внешний угол равен .

93
Многоугольник называется вписанным
в окружность, если все его вершины лежат на
некоторой окружности, которая называется опи-
санной около многоугольника.
Многоугольник называется описанным
около окружности, если все его стороны касают-
ся некоторой окружности, которая, в свою оче-
редь, называется вписанной в многоугольник.

• В любом вписанном в окружность четырёхугольнике сумма


противоположных углов равна . Верно и обратное: если сумма про-
тивоположных углов четырёхугольника равна , то около него можно
описать окружность.
• У четырехугольника, описанного около окружности, суммы
длин противолежащих сторон равны. Верно и обратное: если в выпук-
лом четырехугольнике суммы длин противоположных сторон равны меж-
ду собой, то в него можно вписать окружность.
Теорема Птолемея. Произведение диагоналей вписанного четы-
рёхугольника равно сумме произведений противоположных сторон.
Апофемой правильного многоугольника называют перпендику-
ляр, проведенный из центра правильного многоугольника к его стороне.

94
Отношение площадей двух подобных многоугольников равно
квадрату коэффициента подобия (который, равен отношению их перимет-
ров, или же сходственных сторон).
Площадь правильного n-угольника, описанного окружностью
радиуса и с длиной стороны , вычисляется по формуле:

Периметр правильного многоугольника, вписанного в окруж-


ность вычисляется по формуле: . Длину
стороны можно найти по формуле:
Радиусы описанной ( ) и вписанной ( ) окружностей (вокруг пра-
вильного n-угольника с длиной стороны ) вычисляются по формулам:

Формула Пика (или теорема Пика) — клас-


сический результат комбинаторной геометрии и гео-
метрии чисел, даёт выражение для площади много-
угольника с целочисленными вершинами. Названа в
честь Георга Пика, доказавшего её в 1899 году.
Площадь многоугольника с целочисленными
вершинами равна:

где В – количество целочисленных точек внутри многоугольника,


Г – количество целочисленных точек на границе многоугольника.

ТЕМА 14. ДЕКАРТОВЫ КООРДИНАТЫ НА ПЛОСКОСТИ


Декартова система координат на плоскости создается двумя вза-
имно перпендикулярными осями (ось – ось абсцисс и ось – ось ор-
динат), которые имеют общее начало (начало координат) и одинако-
вый масштаб осей.
Каждой точке плоскости по определенному правилу задаётся в со-
ответствии пара чисел – абсцисса и ордината ( ). Эти числа называются
декартовыми координатами точки.

95
Координатные четверти

Расстояние между двумя точками равно квадратному корню из


суммы квадратов разницы одноименных координат.
√( ) ( )
где – расстояние между точками ( )и ( ).
Координаты середины отрезка
Координаты середины отрезка равны полусумме соответственных
координат его концов.

где ( )и( ) – координаты начала и конца отрезка.


Уравнение фигуры
Уравнением фигуры в декартовых координатах на плоскости
называется уравнение с двумя переменными и , которые удовлетворяют
координатам любой точки фигуры.
Уравнение окружности. Уравнение ( ) ( ) яв-
ляется уравнением окружности с центром в точке ( ) и радиусом .
Если центром окружности является начало координат, то уравне-
ние окружности имеет вид: .
Уравнение прямой. Любая прямая в декартовых координатах и
имеет уравнение вида: , где – любые числа.

96
Уравнение прямой, проходящей через данную точку
( ) в данном направлении, определяемом угловым коэффициен-
том и вычисляется по формуле:
( )
Это уравнение определяет пучок прямых, проходящих через точ-
ку ( ), которая называется центром пучка.
Угловой коэффициент прямой, проходящей через две точки,
определяется по формуле:

Угловой коэффициент прямой равен тангенсу угла наклона пря-


мой. Это угол между данной прямой и осью .
Каноническое уравнение прямой , проходящей через точки
( )и ( ):

Углом между прямыми и определяется по формуле:

Условие перпендикулярности (ортогональности) двух прямых.


Если прямые и заданы соответственно уравнениями и
, то они перпендикулярны .
Уравнение касательной, проведенной в точке к графику
функции ( ):
( ) ( )( ) ( ) ( )( )
Чтобы написать уравнение касательной, нам достаточно знать
уравнение функции и точку, в которой проведена касательная. Тогда мы
сможем найти ( ) и ( ).

ТЕМА 15. ВЕКТОРЫ НА ПЛОСКОСТИ


Многие физические величины (такие как перемещение, сила и
скорость) характеризуются не только своим числовым значением, но и

97
направлением в пространстве. Такие физические величины называются
векторными величинами (векторами). Т.о., вектор – это направлен-
ный отрезок, для которого указано, какая из его граничных точек считает-
ся началом, а какая – концом.
Любая точка плоскости является нулевым вектором (начало та-
кого вектора совпадает с его концом). Обозначение вектора: ⃗ или ⃗⃗⃗⃗⃗⃗ .

Абсолютной величиной (длиной, или модулем) ненулевого век-


тора ⃗⃗⃗⃗⃗⃗ называется длина отрезка . Обозначение: |⃗⃗⃗⃗⃗⃗ |. Абсолютная
величина нуль-вектора равна нулю. Формула вычисления модуля вектора с
координатами :

| ̅| √

Ненулевые векторы ⃗ и ⃗⃗ называются коллинеарными, если они


лежат либо на одной прямой, либо на параллельных прямых. Нулевой век-
тор коллинеарен любому вектору.
Если два ненулевых вектора ⃗ и ⃗⃗ коллинеарны, то они могут
быть направлены одинаково, либо противоположно. Тогда в первом случае
они являются сонаправленными ( ), а во втором – противоположно
направленными ( ).
Векторы называются равными, если они сонаправлены и их дли-
ны равны.
Расстояние между двумя точками вычисляется по формулам:
1) ( ) ( ) | | √( ) ( ) ;

98
2) ( ) ( ) | | √( ) ( ) ( ) .
Скалярным произведением двух векторов ⃗ и ⃗⃗ называется ска-
лярная величина, равная произведению модулей этих векторов умножен-
ного на косинус угла между ними:
⃗⃗ ⃗⃗ |⃗⃗| |⃗⃗|

Действия над векторами в координатном представлении


1) Сложение (вычитание) векторов:
⃗⃗ ⃗⃗ ⃗⃗, где ⃗⃗ { }: , .
2) Умножение вектора на число:
⃗⃗ ⃗⃗, где ⃗⃗ { }.
3) Скалярное произведение двух векторов ⃗ и ⃗⃗ – это скалярная
величина, равная сумме попарного произведения координат векторов
⃗ и ⃗⃗ .
⃗⃗ ⃗⃗
Если векторы ⃗⃗ и ⃗⃗ перпендикулярны, то их скалярное произ-
ведение равно нулю. И обратно, если скалярное произведение ненулевых
векторов ⃗ ⃗⃗ , то векторы ⃗ и ⃗⃗ – перпендикулярны.
Сумма и разность векторов

Три вектора называются компланарными, если они, будучи при-


ведёнными к общему началу, лежат в одной плоскости (т.е. векторы, кото-
рые параллельны одной плоскости или лежат на одной плоскости, назы-
ваются компланарными векторами).
Всегда возможно найти плоскость, параллельную двум произ-
вольным векторам, поэтому любые два вектора всегда компланарные.

99
Eсли из трёх векторов два коллинеарны, то очевидно, что эти три
вектора компланарны.
Признак компланарности трёх векторов
Пусть векторы ⃗ и ⃗⃗ не коллинеарны. Если для вектора ⃗ суще-
ствует единственная пара действительных чисел и , такая, что вектор
⃗ ⃗ ⃗⃗ , то векторы ⃗, ⃗⃗ и ⃗ компланарны.

ТЕМА 16. ВВЕДЕНИЕ В СТЕРЕОМЕТРИЮ. АКСИОМЫ СТЕ-


РЕОМЕТРИИ
Стереометрия – это раздел геометрии, в котором изучаются фи-
гуры в пространстве. Основными фигурами стереометрии являются точка,
прямая и плоскость.
Аксиома (постулат) – утверждение, принимаемое без доказатель-
ства (в отличие от теоремы). Аксиомы стереометрии – основные свойства
основных фигур стереометрии. Точка и прямая – основные фигуры плани-
метрии, поэтому в стереометрии справедливы аксиомы планиметрии.
Аксиомы стереометрии

100
Следствия из аксиом стереометрии
1. Через прямую и точку, не лежащую на ней, можно повести
плоскость, причем только одну.
2. Если две точки прямой принадлежат плоскости, то и вся прямая
принадлежит этой плоскости.
3. Плоскость и прямая, не лежащая на ней, либо не пересекаются,
либо пересекаются в одной точке.
4. Через три точки, не лежащие на одной прямой, можно провести
плоскость, причем только одну.

ТЕМА 17. МНОГОГРАННИКИ


Геометрическое тело – часть пространства, которое занимает
предмет. Примеры пространственных геометрических тел:
 Куб – это тело, поверхность которого ограничена шестью равными
квадратами (рис. 1).
 Прямоугольный параллелепипед – это тело, поверхность которого
ограничена шестью прямоугольниками (рис. 2).
 Тетраэдр (треугольная пирамида) – это тело, поверхность которого
ограничена четырьмя треугольниками (рис. 3).
 Правильным тетраэдром называется тело, поверхность которого
ограничена четырьмя равными правильными треугольниками (равно-
сторонними).
 Многогранник, или полиэдр — обычно замкнутая поверхность, со-
ставленная из многоугольников, но иногда также называют тело, огра-
ниченное этой поверхностью. Многоугольники, которые ограничива-
ют поверхность тела, называются гранями, стороны граней – рёбрами,
вершины граней – вершинами многогранника (рис. 4).
 Призма ( -угольная) – это многогранник, у которого две грани – рав-
ные n-угольники, которые лежат в параллельных плоскостях, а другие
n граней – параллелограммы (рис.5).
Многоугольники называются основаниями призмы, а параллело-
граммы – боковыми гранями. Стороны боковых граней и оснований назы-
ваются ребрами призмы. Концы ребер – вершинами призмы. Боковыми
рёбрами называются рёбра, которые не принадлежат основаниям.

101
Рисунок 1. Рисунок 2. Рисунок 3.

Рисунок 4. Рисунок 5.
Свойства призмы
1. Основания призмы параллельны и равны.
2. Боковые рёбра параллельны и равны.
3. Боковые грани – параллелограммы.
Высотой призмы называется перпендикуляр, проведенный из точ-
ки верхнего основания на плоскость нижнего основания.
Диагональю призмы называется отрезок, который соединяет две
вершины, которые не лежат на одной грани.
Прямой призмой называется призма, у которой боковые ребра
перпендикулярны площади оснований. Призма, которая не является пря-
мой, называется наклонной.
Параллелепипедом называется призма, основания которой – па-
раллелограммы.
Свойства параллелепипеда
1. Противоположные грани параллелепипеда попарно равны и па-
раллельны.
2. Все диагонали параллелепипеда пересекаются в одной точке и
делятся ею пополам.

102
Параллелепипед называется прямым, если у него боковые рёбра
перпендикулярны основаниям. Прямой параллелепипед, основаниями ко-
торого являются прямоугольники, называются прямоугольным. Все грани
прямоугольного параллелепипеда – прямоугольники.
Свойства прямоугольного параллелепипеда
1. Все диагонали равны.
2. Квадрат диагонали равен сумме квадратов трёх его измерений.
Прямоугольный параллелепипед, у которого все рёбра равны,
называется кубом.

ТЕМА 18. ПИРАМИДА. ЕЁ ВИДЫ И СВОЙСТВА


Пирамида (n-угольная) – это многогранник, у которого одна грань
является произвольным n-угольником, а другие n грани – треугольники,
которые имеют общую вершину; n-угольник называется основанием, а
треугольники – боковыми гранями. Общая вершина боковых граней назы-
вается вершиной пирамиды.
Высотой пирамиды называется перпендикуляр, проведенный из
вершины пирамиды на плоскость основания.
Высота боковой грани правильной пирамиды, проведенная из
вершины пирамиды, называется апофемой.

На рисунке изображено: A – вершина пирамиды, AB, AC, AD, AE


– рёбра пирамиды, ADE, AEB, ABC, ACD – боковые грани пирамиды, AG
– высота, AF – апофема, AEC – диагональное сечение.

103
Правильной называется пирамида, в основании которой лежит
правильный многоугольник, а основание высоты пирамиды совпадает с
центром этого многоугольника.
У правильной пирамиды:
 боковые грани и рёбра равны;
 апофемы равны;
 двугранные углы при основании и боковых рёбрах равны;
 каждая точка высоты правильной пирамиды равноудалена от всех
вершин основания, всех боковых граней и от всех боковых рёбер.
Диагональным сечением пирамиды называется сечение плоско-
стью, которая проходит через два боковых ребра пирамиды, которые не
принадлежат одной грани.
Теорема. Если пирамида пересекается плоскостью, параллельно
основанию, то:
1) боковые рёбра и высота пирамиды делится этой плоскостью
на пропорциональные части;
2) сечение – многоугольник, подобный основанию;
3) площади сечения и основания относятся как два квадрата их
расстояний от вершины пирамиды.
Высотой усеченной пирамиды называется перпендикуляр, прове-
денный из любой точки одного основания на плоскость другого основа-
ния.
Усеченная пирамида называется правильной, если она составляет
часть правильной пирамиды. Высота боковой грани правильной усеченной
пирамиды, проведенная к ребру основания, называется апофемой.
В правильной усечённой пирамиде:
 боковые рёбра и боковые грани равны;
 апофемы равны;
 двугранные углы при каждом основании равны;
 двугранные углы при боковых рёбрах равны;
 каждая точка прямой, которая проходит через центры её основа-
ний, равноудалённая от всех вершин каждого основания, равно-
удалена от плоскостей боковых граней и равноудалена от прямых,
на которых лежат боковые рёбра.

104
ТЕМА 19. ПЛОЩАДИ И ОБЪЕМЫ МНОГОГРАННИКОВ
Проще всего найти объём куба – это куб его стороны.
Объём параллелепипеда – произведение длины, ширины и высоты.
Объём призмы – произведение площади её основания на высоту.
Объём пирамиды – это треть произведения площади основания на
высоту.
Площадь боковой поверхности прямой (наклонной) призмы –
произведение периметра основания на высоту (длину наклонной).

105
ТЕМА 20. ТЕЛА ВРАЩЕНИЯ: ЦИЛИНДР И КОНУС. ШАР И
СФЕРА. ПЛОЩАДИ И ОБЪЁМЫ
Цилиндром называется тело, созданное вращением
прямоугольника вокруг его стороны.
Основания цилиндра – равные круги, которые ле-
жат в параллельных плоскостях. Радиусы кругов называют
радиусами цилиндра. Сторона CD описывает поверхность,
которая называется боковой поверхностью цилиндра.
Высота цилиндра – отрезок, перпендикулярный
основанию, концы которого
принадлежат основаниям. Высота цилиндра
равна его образующей.
Осевое сечение – прямоугольник со
сторонами, который равен высоте цилиндра
и диаметру его основания.
Сечение цилиндра плоскостью,
перпендикулярно его осе – это круг, кото-
рый равен основанию, а сечение плоско-
стью, параллельно оси, – прямоугольник
(или отрезок). Виды сечений цилиндра по-
казаны на рисунке слева.
Конусом называется тело, созданное вращением прямоугольного
треугольника вокруг одного из его катетов.
Если прямоугольный тре-
угольник POB вращается вокруг катета
PO, то его гипотенуза PB описывает
боковую поверхность, а катет OB –
круг – основание конуса. Радиус этого
круга называется радиусом конуса;
точка P, отрезок PB, отрезок PO, пря-
мая PO называется соответственно
вершиной, образующей, высотой и
осью конуса.

106
Осевое сечение конуса – сечение конуса плос-
костью, которая проходит через его ось. Все осевые се-
чения конуса являются равнобедренными треугольни-
ками, равными между собой.
Усеченным конусом называется часть конуса,
ограниченная его основанием и сечением, параллельным
площади основания. Такой конус можно получить в ре-
зультате вращения равнобедренной трапеции вокруг её
оси симметрии или вращая прямоугольную трапецию
вокруг оси, которая совпадает с боковой стороной
трапеции, перпендикулярно основаниям.
Осевое сечение усеченного конуса – рав-
нобокая трапеция.
Расстояние между основаниями усечен-
ного конуса – высота этого конуса.
Сфера – поверхность, состоящая из всех точек пространства, ко-
торые находятся на данном расстоянии (которое называется радиусом) от
данной точки (которая называется центром сферы).
Отрезок, который соединяет
две точки сферы и проходит через её
центр, называется диаметром сферы.
На рисунке слева О – центр сферы;
ОВ, ОС, ОК – радиусы; ВС – диаметр
сферы. Сферу можно получить в ре-
зультате вращения круга вокруг его
диаметра.
Шаром называется тело, созданное из всех точек пространства,
которые находятся на расстоянии, не больше данного (которое называется
радиусом) от данной точки (которая называется центром). Шар можно
получить в результате вращения круга вокруг его диаметра.
Сечение шара плоскостью является окружность. Центр круга
(окружности) – основание перпендикуляра, опущенного из центра шара
(сферы) на усеченную плоскость. Сечение, которое проходит через центр
шара (сферы), называется большим кругом (окружностью).

107
Плоскость (прямая), которая имеет с шаром (сферой) только одну
общую точку, называется касательной плоскостью (прямой). Касатель-
ная плоскость (прямая) перпендикулярна радиусу шара (сферы), прове-
денная в точку касания. Если плоскость (прямая) проходит через точку
сферы и перпендикулярна радиусу, который проведён в эту точку, то она
касается сферы.
Шаровым сегментом называется тело, отсеченное от шара плос-
костью. Шаровой сегмент, ограниченный кругом называют основанием, и
сферическим сегментом. Отрезок диаметра,
перпендикулярный основанию шарового (сфе-
ричного) сегмента, который содержится между
основанием и сферой, называется высотой
шарового (сферического) сегмента.
Площади и объёмы цилиндра, конуса и шара

ТЕМА 21. ДЕКАРТОВЫ КООРДИНАТЫ В ПРОСТРАНСТВЕ


Декартова система координат в пространстве задаётся тремя
попарно перпендикулярными осями (ОХ – ось абсцисс, ОY – ось ординат и

108
OZ – ось аппликат), которые имеют общее начало О (начало координат) и
одинаковый масштаб вдоль осей.
Каждой точке пространства по определённому правилу задаётся
соответственная троица чисел – абсцисса, ордината, аппликата (x; y; z),
которые наз. декартовыми координатами точки.

109
ПРИЛОЖЕНИЕ

Приложение содержит ряд полезных школьных таблиц по матема-


тике, а также примеры и решения задач из ОГЭ и ЕГЭ, вызывающие труд-
ности у обучающихся при подготовке к экзаменам.

Таблица классов и разрядов Таблица Пифагора

Таблица квадратов

110
111
112
113
114
СТАТИСТИКА, ТЕОРЕМЫ О ВЕРОЯТНОСТНЫХ СОБЫТИЯХ
(ЗАДАЧИ ИЗ ОГЭ И ЕГЭ)
Задача 1. Записан рост (в сантиметрах) пяти учащихся: 158, 166,
134, 130, 132. На сколько отличается среднее арифметическое этого набора
чисел от его медианы?
Решение. Медианой ряда, состоящего из нечетного количества чи-
сел, называется число данного ряда, которое окажется посередине, если
этот ряд упорядочить.
Медианой ряда, состоящего из чётного количества чисел, называ-
ется среднее арифметическое двух стоящих посередине чисел этого ряда.
Упорядочим данный ряд: 130, 132, 134, 158, 166, следовательно,
медиана равна 134. Среднее арифметическое же будет равно:

Разница между медианой и средним арифметическим составляет


144 – 134 = 10.
О т в е т : 10.
Задача 2. Фирма «Вспышка» изготавливает фонарики. Вероят-
ность того, что случайно выбранный фонарик из партии бракованный,
равна 0,02. Какова вероятность того, что два случайно выбранных из од-
ной партии фонарика окажутся небракованными?
Решение. Вероятность того, что один случайно выбранный из пар-
тии фонарик — небракованный, составляет 1 − 0,02=0,98. Вероятность
того, что мы выберем одновременно два небракованных фонарика равна
0,98 · 0,98 = 0,9604.
Ответ: 0,9604.
Задача 3. Средний рост жителя города, в котором живет Даша, ра-
вен 170 см. Рост Даши 173 см. Какое из следующих утверждений верно?
1) Даша — самая высокая девушка в городе.
2) Обязательно найдется девушка ниже 170 см.
3) Обязательно найдется человек ростом менее 171 см.
4) Обязательно найдется человек ростом 167 см.
Решение. Первое утверждение неверно: например, в городе могут
жить три девушки ростом 162 см, 173 см и и 175 см.

115
Второе утверждение неверно: в городе может жить только одна
девушка – Даша.
Третье утверждение верно: если все жители будут не ниже 171 см,
то средний рост будет не меньше 171 см.
Четвёртое утверждение неверно: например, в городе могут жить
трое жителей ростом 165 см, 172 см и 173 см.
О т в е т : 3.
Задача 4. Известно, что в некотором регионе вероятность того,
что родившийся младенец окажется мальчиком, равна 0,512. В 2010 г. в
этом регионе на 1000 родившихся младенцев в среднем пришлось 477 де-
вочек. Насколько частота рождения девочек в 2010 г. в этом регионе отли-
чается от вероятности этого события?
Решение. Частота cобытия «рождение девочки» равна
477 : 1000 = 0,477. Вероятность рождения девочки в этом регионе равна
1 − 0,512 = 0,488. Поэтому частота данного события отличается от его ве-
роятности на 0,488 − 0,477 = 0,011.
О т в е т : 0,011.
Задача 5. Вероятность того, что новая шариковая ручка пишет
плохо (или не пишет), равна 0,19. Покупатель в магазине выбирает одну
такую ручку. Найдите вероятность того, что эта ручка пишет хорошо.
Решение. Вероятность того, что ручка пишет хорошо равна
1 − 0,19 = 0,81.
О т в е т : 0,81.
Задача 6. На экзамене по геометрии школьнику достаётся одна за-
дача из сборника. Вероятность того, что эта задача по теме «Углы», равна
0,1. Вероятность того, что это окажется задача по теме«Параллелограмм»,
равна 0,6. В сборнике нет задач, которые одновременно относятся к этим
двум темам. Найдите вероятность того, что на экзамене школьнику доста-
нется задача по одной из этих двух тем.
Решение. Суммарная вероятность несовместных событий равна
сумме вероятностей этих событий: .
О т в е т : 0,7.
Задача 7. Игральную кость бросают дважды. Найдите вероятность
того, что сумма двух выпавших чисел равна 4 или 7.

116
Решение. Сумма двух выпавших чисел будет равна 4 в трех случа-
ях(1 и 3, 3 и 1, 2 и 2) и 7 в шести случаях(1 и 6, 6 и 1, 2 и 5, 5 и 2, 3 и 4, 4 и
3), т. е. 9 благоприятных событий. А всего событий может быть 6 · 6 = 36,
значит, вероятность равна .
О т в е т : 0,25.
КЛАССИЧЕСКИЕ ВЕРОЯТНОСТИ (ИЗ ОГЭ И ЕГЭ)
Задача 1. На экзамене 25 билетов, Сергей не выучил 3 из них.
Найдите вероятность того, что ему попадётся выученный билет.
Решение. Сергей выучил 25 − 3 = 22 вопроса. Поэтому вероят-
ность того, что ему попадётся выученный билет равна .
О т в е т : 0,88.
Задача 2. Коля выбирает трёхзначное число. Найдите вероятность
того, что оно делится на 5.
Решение. Всего 3-хзначных чисел 900. На 5 делится каждое пятое
их них, то есть таких чисел . Вероятность того, что Коля выбрал
3-хзначное число, делящееся на 5, равно отношению 3-хзначных чисел, де-
лящихся на 5, ко всему количеству 3-хзначных чисел: .
О т в е т : 0,2.
Задача 3. Телевизор у Маши сломался и показывает только один
случайный канал. Маша включает телевизор. В это время по трем каналам
из двадцати показывают кинокомедии. Найдите вероятность того, что
Маша попадет на канал, где комедия не идет.
Решение. Количество каналов, по которым не идет кинокоме-
дий . Вероятность того, что Маша не попадет на канал, по ко-
торому идут кинокомедии равна отношению количества каналов, по кото-
рым не идут кинокомедии к общему числу каналов: .
О т в е т : 0,85.
Задача 4. На тарелке 12 пирожков: 5 с мясом, 4 с капустой и 3 с
вишней. Наташа наугад выбирает один пирожок. Найдите вероятность
того, что он окажется с вишней.

117
Решение. Вероятность того, что будет выбран пирожок с вишней
равна отношению количества пирожков с вишней к общему количеству
пирожков: .
О т в е т : 0,25
Задача 5. В фирме такси в данный момент свободно 20 машин: 9
чёрных, 4 жёлтых и 7 зелёных. По вызову выехала одна из машин, случай-
но оказавшаяся ближе всего к заказчику. Найдите вероятность того, что к
нему приедет жёлтое такси.
Решение. Вероятность того, что приедет жёлтая машина равна от-
ношению жёлтых машин к общему количеству всех машин: .
О т в е т : 0,2.
Задача 6. В каждой десятой банке кофе согласно условиям акции
есть приз. Призы распределены по банкам случайно. Варя покупает банку
кофе в надежде выиграть приз. Найдите вероятность того, что Варя не най-
дет приз в своей банке.
Решение. Так как в каждой десятой банке кофе есть приз, то веро-
ятность выиграть приз равна . Поэтому, вероятность не выиграть приз
равна .
О т в е т : 0,9.
Задача 7. Миша с папой решили покататься на колесе обозрения.
Всего на колесе двадцать четыре кабинки, из них 5 — синие, 7 — зеленые,
остальные — красные. Кабинки по очереди подходят к платформе для по-
садки. Найдите вероятность того, что Миша прокатится в красной кабинке.
Решение. Вероятность того, что подойдет красная кабинка равна
отношению количества красных кабинок к общему количеству кабинок на
колесе обозрения. Всего красных кабинок: . Поэтому иско-
мая вероятность равна .
О т в е т : 0,5.
Задача 8. У бабушки 20 чашек: 5 с красными цветами, остальные
с синими. Бабушка наливает чай в случайно выбранную чашку. Найдите
вероятность того, что это будет чашка с синими цветами.
Решение. Вероятность того, что чай нальют в чашку с синими цве-
тами равна отношению количества чашек с синими цветами к общему ко-

118
личеству чашек. Всего чашек с синими цветами: . Поэтому ис-
комая вероятность .
О т в е т : 0,75.
Задача 9. Родительский комитет закупил 25 пазлов для подарков
детям на окончание года, из них 15 с машинами и 10 с видами городов.
Подарки распределяются случайным образом. Найдите вероятность того,
что Толе достанется пазл с машиной.
Решение. Вероятность получить пазл с машиной равна отношению
числа пазлов с машиной к общему числу всех пазлов, то есть .
О т в е т : 0,6.
Задача 10. В среднем из каждых 80 поступивших в продажу акку-
муляторов 76 аккумуляторов заряжены. Найдите вероятность того, что
купленный аккумулятор не заряжен.
Решение. Из каждых 80 аккумуляторов в среднем будет
незаряженных. Таким образом, вероятность купить незаряженный
аккумулятор равна доле числа незаряженных аккумуляторов из каждых 80
купленных, то есть .
О т в е т : 0,05.
Задача 11. Какое из следующих чисел не является членом данной
( )
последовательности, если она задана формулой .

) ) ) )
Решение. Найдём несколько первых членов последовательности:
— число, указанное под номером 1),
— число, указанное под номером 4),
– число, указанное под номером 3), , , ,
.
Покажем, что число указанное под номером 2, не является чле-
ном последовательности. Действительно, первые 8 членов последователь-
ности уже проверены. При первое слагаемое в сумме

119
( )
не меньше 9, а абсолютная величина второго слагаемого не больше
. Поэтому для всех справедлива оценка .
О т в е т : 2.
Задача 12. Для экзамена подготовили билеты с номерами от 1 до
50. Какова вероятность того, что наугад взятый учеником билет имеет од-
нозначный номер?
Решение. Всего было подготовлено 50 билетов. Среди них 9 были
однозначными. Таким образом вероятность того, что наугад взятый учени-
ком билет имеет однозначный номер равна .
О т в е т : 0,18.
Задача 13. В мешке содержатся жетоны с номерами от 5 до 54
включительно. Какова вероятность, того, что извлеченный наугад из меш-
ка жетон содержит двузначное число?
Решение. Всего в мешке жетонов - 50. Среди них 45 имеют дву-
значный номер. Таким образом, вероятность, того, что извлеченный наугад
из мешка жетон содержит двузначное число равна .
О т в е т : 0,9.
Задача 14. В денежно-вещевой лотерее на 100 000 билетов разыг-
рывается 1300 вещевых и 850 денежных выигрышей. Какова вероятность
получить вещевой выигрыш?
Решение. Вероятность получить вещевой выигрыш равна отноше-
нию количества вещевых выйграшей к общему количеству выйгрышей
.
О т в е т : 0,013.
Задача 15. Из 900 новых флеш-карт в среднем 54 не пригодны для
записи. Какова вероятность того, что случайно выбранная флеш-карта при-
годна для записи?
Решение. Из 900 карт исправны 900 − 54 = 846 шт. Поэтому веро-
ятность того, что случайно выбранная флеш-карта пригодна для записи
равна: .
О т в е т : 0,94.

120
Задача 16. В чемпионате по футболу участвуют 16 команд, кото-
рые жеребьевкой распределяются на 4 группы: A, B, C и D. Какова вероят-
ность того, что команда России не попадает в группу A?
Решение. Каждая команда попадет в группу с вероятностью 0,25.
Таким образом, вероятность того, что команда не попадает в группу равна
.
О т в е т : 0,75.
Задача 17. В группе из 20 российских туристов несколько человек
владеют иностранными языками. Из них пятеро говорят только по-
английски, трое только по-французски, двое по-французски и по-
английски. Какова вероятность того, что случайно выбранный турист гово-
рит по-французски?
Решение. Количество туристов, говорящих по-французски, равно
5. Поэтому вероятность того, что случайно выбранный турист говорит по-
французски равна .
О т в е т : 0,25.
Задача 18. В коробке 14 пакетиков с чёрным чаем и 6 пакетиков с
зелёным чаем. Павел наугад вынимает один пакетик. Какова вероятность
того, что это пакетик с зелёным чаем?
Решение. Всего в коробке 14+6=20 пакетиков. Вероятность того,
что Павел вытащит пакетик с зелёным чаем равна .
О т в е т : 0,3.
Задача 19. Стас, Денис, Костя, Маша, Дима бросили жребий —
кому начинать игру. Найдите вероятность того, что начинать игру должна
будет девочка.
Решение. Вероятность события равна отношению количества бла-
гоприятных случаев к количеству всех случаев. Среди пяти детей одна
девочка. Поэтому вероятность равна .
Ответ: 0,2.
Задача 20. Перед началом футбольного матча судья бросает мо-
нетку, чтобы определить, какая из команд будет первой владеть мячом.
Команда А должна сыграть два матча — с командой В и с командой С.

121
Найдите вероятность того, что в обоих матчах первой мячом будет владеть
команда А.
Решение. Рассмотрим все возможные исходы жеребьёвки.
Команда А в матче в обоих матчах первой владеет мячом.
Команда А в матче в обоих матчах не владеет мячом первой.
Команда А в матче с командой В владеет мячом первой, а в матче
с командой С — второй.
Команда А в матче с командой С владеет мячом первой, а в матче
с командой В — второй.
Из четырех исходов один является благоприятным, вероятность
его наступления равна 0,25.
О т в е т : 0,25.
Задача 21. В лыжных гонках участвуют 11 спортсменов из Рос-
сии, 6 спортсменов из Норвегии и 3 спортсмена из Швеции. Порядок, в ко-
тором спортсмены стартуют, определяется жребием. Найдите вероятность
того, что первым будет стартовать спортсмен из России.
Решение. Всего спортсменов. Поэтому вероят-
ность того, что стартовать будет спортсмен из России равна .
О т в е т : 0,55.
Задача 22. В лыжных гонках участвуют 11 спортсменов из Рос-
сии, 6 спортсменов из Норвегии и 3 спортсмена из Швеции. Порядок, в ко-
тором спортсмены стартуют, определяется жребием. Найдите вероятность
того, что первым будет стартовать спортсмен не из России.
Решение. Всего спортсменов . Поэтому вероят-
ность того, что стартовать будет спортсмен не из России равна .
О т в е т : 0,45.
Задача 23. Из каждых 1000 электрических лампочек 5 бракован-
ных. Какова вероятность купить исправную лампочку?
Решение. Вероятность купить исправную лампочку равна доле ис-
правных лампочек в общем количестве лампочек: .
О т в е т : 0,995.

122
Задача 24. Петя, Вика, Катя, Игорь, Антон, Полина бросили жре-
бий — кому начинать игру. Найдите вероятность того, что начинать игру
должен будет мальчик.
Решение. Вероятность события равна отношению количества бла-
гоприятных случаев к количеству всех случаев. Благоприятными случаями
являются 3 случая, когда игру начинает Петя, Игорь или Антон, а количе-
ство всех случаев 6. Поэтому искомое отношение равно .
О т в е т : 0,5.
Задача 25. Из 1600 пакетов молока в среднем 80 протекают. Како-
ва вероятность того, что случайно выбранный пакет молока не течёт?
Решение. Вероятность того, что пакет молока протекает
на . Поэтому вероятность того, что случайно выбранный
пакет молока не течёт равна .
О т в е т : 0,95.
Задача 26. В соревнованиях по художественной гимнастике участ-
вуют 3 гимнастки из России, 3 гимнастки из Украины и 4 гимнастки из Бе-
лоруссии. Порядок выступлений определяется жеребьёвкой. Найдите веро-
ятность того, что первой будет выступать гимнастка из России.
Решение. Всего в соревнованиях участвуют гимна-
сток. Поэтому вероятность того, что первой будет будет выступать гим-
настка из России равна .
О т в е т : 0,3.
Задача 27. Определите вероятность того, что при бросании иг-
рального кубика (правильной кости) выпадет нечетное число очков.
Решение. При бросании кубика равновозможны шесть различных
исходов. Событию "выпадет нечётное число очков" удовлетворяют три
случая: когда на кубике выпадает 1, 3 или 5 очков. Поэтому вероятность
того, что на кубике выпадет нечётное число очков равна .
О т в е т : 0,5.
Задача 28. Определите вероятность того, что при бросании кубика
выпало число очков, не большее 3.
Решение. При бросании кубика равновозможны шесть различных
исходов. Событию "выпадет не больше трёх очков" удовлетворяют три

123
случая: когда на кубике выпадает 1, 2, или 3 очка. Поэтому вероятность
того, что на кубике выпадет не больше трёх очков равна .
О т в е т : 0,5.
Задача 29. В случайном эксперименте симметричную монету бро-
сают дважды. Найдите вероятность того, что орел выпадет ровно 1 раз.
Решение. Всего возможны четыре исхода: решка-решка, решка-
орёл, орёл-решка, орёл-орёл. Орёл выпадает ровно один раз в двух случа-
ях, поэтому вероятность того, что орёл выпадет ровно один раз: .
О т в е т : 0,5.
Задача 30. Игральную кость бросают дважды. Найдите вероят-
ность того, что оба раза выпало число, большее 3.
Решение. При бросании кубика равновозможны шесть различных
исходов. Событию "выпадет больше трёх очков" удовлетворяют три слу-
чая: когда на кубике выпадает 4, 5, или 6 очков. Поэтому вероятность того,
что на кубике выпадет не больше трёх очков равна . Таким образом,
при одном бросании кубика с одинаковой вероятностью реализуется либо
событие А — выпало число, большее 3, либо событие Б — выпало число
не больше 3. То есть равновероятно реализуютсячетыре события: А-А, А-
Б, Б-А, Б-Б. Поэтому вероятность того, что оба раза выпало число, боль-
шее 3 равна .
О т в е т : 0,25.
Задача 31. Стрелок 4 раза стреляет по мишеням. Вероятность по-
падания в мишень при одном выстреле равна 0,5. Найдите вероятность
того, что стрелок первые 3 раза попал в мишени, а последний раз промах-
нулся.
Решение. Вероятность промаха равна 1 − 0,5 = 0,5. Вероятность
того, что стрелок первые три раза попал в мишени равна 0,53 = 0,125. От-
куда, вероятность события, при котором стрелок сначала три раза попадает
в мишени, а четвёртый раз промахивается равна 0,125 · 0,5 = 0,0625.
О т в е т : 0,0625.
Задача 32. В таблице представлены результаты четырёх стрелков,
показанные ими на тренировке. Тренер решил послать на соревнования

124
того стрелка, у которого относительная частота попаданий выше. Кого из
стрелков выберет тренер? Укажите в ответе его номер.

Номер стрелка Число выстрелов Число попаданий


1 42 28

2 70 20

3 54 45

4 46 42

Решение. Найдём относительную частоту попаданий каждого из


стрелков:
Заметим, что . Приведём и к общему знаменателю
и сравним: . Таким образом, наибольшая относительная
частота попаданий у четвёртого стрелка.
О т в е т : 4.
Задача 33. В магазине канцтоваров продаётся 100 ручек, из них 37
– красные, 8 – зелёные, 17 – фиолетовые, ещё есть синие и чёрные, их по-
ровну. Найдите вероятность того, что Алиса наугад вытащит красную или
чёрную ручку.
Решение. Найдём количество чёрных ручек: . Ве-
роятность того, что Алиса вытащит наугад красную или чёрную ручку
равна .
О т в е т : 0,56.
Задача 34. В среднем из 100 карманных фонариков, поступивших
в продажу, восемь неисправных. Найдите вероятность того, что выбран-
ный наудачу в магазине фонарик окажется исправен.
Решение. Из 100 фонариков исправны. Значит, ве-
роятность того, что выбранный наудачу в магазине фонарик окажется од-
ним из них равна ( )
О т в е т : 0,92.

125
Задача 35. Вероятность того, что новая батарейка бракованная,
равна 0,04 (независимо от других батареек). Покупатель в магазине выби-
рает случайную упаковку, в которой две таких батарейки. Найдите вероят-
ность того, что ровно одна батарейка в упаковке окажется исправной.
Решение. Введем два события:
А: «первая батарейка в упаковке бракованная» и B: «вторая бата-
рейка в упаковке бракованная».
По условию задания эти события независимы (наступление одно-
го из них не изменяет вероятность наступления другого).
Вероятность брака только первой батарейки, равна P(A)∙[1-P(B)], а
вероятность брака только второй батарейки: [1-P(A)]∙ P(B).
Нас интересует возникновение ИЛИ первого исхода ИЛИ второго
исхода. (Союз ИЛИ в теории вероятностей соответствует сложению веро-
ятностей).
Получаем (для несовместных исходов):
( ) ( ) ( ) ( )
( ) ( )
О т в е т : 0,0768.
Задание 36. Гигрометр измеряет влажность в помещении картин-
ной галереи. Вероятность того, что влажность окажется выше 40%, равна
0,82. Вероятность того, что влажность окажется ниже 56 %, равна 0,74.
Найдите вероятность того, что влажность находится в пределах от 40 % до
56 %.
Решение. Введем два события:
: «влажность окажется не выше 40%» и : «влажность окажется в
пределах от 40% до 56 %»
Тогда, сумма этих двух событий будет означать «влаж-
ность окажется ниже 56%».
Вероятность события , равна ( ) , а вероят-
ность события ( ) . Учитывая несовместность событий (не могут
проявиться одновременно) и , имеем:
( ) ( )
( ) ( ) ( )
( ) ( ) ( )

126
( )
О т в е т : 0,56.
Задача 37. За круглый стол на 21 стул в случайном порядке рас-
саживаются 19 мальчиков и 2 девочки. Найдите вероятность того, что де-
вочки не окажутся на соседних местах.
Решение. Решим эту задачу от обратного, сначала найдем вероят-
ность того, что девочки окажутся рядом, а затем, вычислим обратную ве-
роятность по формуле .
Допустим, первая девочка уже куда-то села (вероятность этого со-
бытия ). Осталось мест и вторая девочка должна сесть или слева или
справа от нее. Имеем благоприятное число исходов и общее число
исходов :

тогда противоположная вероятность того, что девочки не будут сидеть


рядом, равна:

О т в е т : 0,9.
Задача 38. Два автомобилиста, независимо друг от друга, выез-
жают из пункта в пункт . Навигатор предлагает каждому из них 10
равноценных маршрутов, и автомобилисты выбирают маршрут случайным
образом. Найдите вероятность того, что автомобилисты выберут различ-
ные маршруты.
Решение. Пусть первый автомобилист выбрал один из 10 маршру-
тов. Следовательно, чтобы второй не выбрал такой же маршрут, его нави-
гатор должен случайным образом выбрать один из 9 оставшихся. Таким
образом, имеем число благоприятных исходов и общее число исхо-
дов :

О т в е т : 0,9.
Задача 39. В торговом центре два одинаковых автомата продают
кофе. Вероятность того, что к концу дня в автомате закончится кофе, равна
0,3. Вероятность того, что кофе закончится в обоих автоматах, равна 0,12.

127
Найдите вероятность того, что к концу дня кофе останется в обоих автома-
тах.
Решение. Рассмотрим события « = кофе закончится в 1-ом авто-
мате» и « = кофе закончится во 2-ом автомате».
Тогда « = кофе закончится в обоих автоматах», « = кофе
закончится хотя бы в одном автомате». По условию ( ) ( ) и
( ) .
События и совместные (появление одного из них не исклю-
чает появление другого в одном и том же испытании), вероятность суммы
двух совместных событий равна сумме вероятностей этих событий,
уменьшенной на вероятность их произведения:
( ) ( ) ( ) ( )
Следовательно, вероятность противоположного события, состоя-
щего в том, что кофе останется в обоих автоматах, равна .
О т в е т : 0,52.
Приведём другое решение. Вероятность того, что кофе останется в
первом автомате равна . Вероятность того, что кофе останет-
ся во втором автомате равна . Вероятность того, что кофе
останется в первом или втором автомате равна . Поскольку
( ) ( ) ( ) ( ), имеем: , откуда
искомая вероятость .
Примечание. Заметим, что события и не являются независи-
мыми. Действительно, вероятность произведения независимых событий
была бы равна произведению вероятностей этих событий: ( )
, однако, по условию, эта вероятность равна .
Задача 40. При артиллерийской стрельбе автоматическая система
делает выстрел по цели. Если цель не уничтожена, то система делает по-
вторный выстрел. Выстрелы повторяются до тех пор, пока цель не будет
уничтожена. Вероятность уничтожения некоторой цели при первом вы-
стреле равна 0,4, а при каждом последующем — 0,6. Сколько выстрелов
потребуется для того, чтобы вероятность уничтожения цели была не менее
0,98? В ответе укажите наименьшее необходимое количество выстрелов.
Решение. Найдем вероятность противоположного события, состо-
ящего в том, что цель не будет уничтожена за n выстрелов. Вероятность

128
промахнуться при первом выстреле равна 0,6, а при каждом следующем —
0,4. Эти события независимые, вероятность их произведения равна произ-
ведению вероятности этих событий. Поэтому вероятность промахнуться
при n выстрелах равна: ( )( ) .
Осталось найти наименьшее натуральное решение неравенства
( )( )
( )
Последовательно проверяя значения , равные 1, 2, 3 и т. д. нахо-
дим, что искомым решением является . Следовательно, необходимо
сделать 5 выстрелов.
О т в е т : 5.
Примечание. Можно решать задачу «по действиям», вычисляя ве-
роятность уцелеть после ряда последовательных промахов:
( )
( ) ( )
( ) ( )
( ) ( )
( ) ( )
Последняя вероятность меньше 0,02, поэтому достаточно пяти вы-
стрелов по мишени.
Приведём другое решение. Вероятность поразить мишень равна
сумме вероятностей поразить её при первом, втором, третьем и т. д. вы-
стрелах. Поэтому задача сводится к нахождению наименьшего натураль-
ного решения неравенства:
( )
( )
В нашем случае неравенство решается подбором, в общем случае
понадобится формула суммы геометрической прогрессии, использование
которой сведет задачу к простейшему логарифмическому неравенству.
Ответ: 5.
Задача 41. Перед началом волейбольного матча капитаны команд
тянут честный жребий, чтобы определить, какая из команд начнёт игру с
мячом. Команда «Статор» по очереди играет с командами «Ротор», «Мо-

129
тор» и «Стартер». Найдите вероятность того, что «Статор» будет начинать
только первую и последнюю игры.
Решение. Требуется найти вероятность произведения трех собы-
тий: «Статор» начинает первую игру, не начинает вторую игру, начинает
третью игру. Вероятность произведения независимых событий равна про-
изведению вероятностей этих событий. Вероятность каждого из них равна
0,5, откуда находим: 0,5·0,5·0,5 = 0,125.
О т в е т : 0,125
ПРИМЕРЫ ИНТЕРЕСНЫХ ТЕКСТОВЫХ ЗАДАЧ
Задача 1. Свежие фрукты содержат 93% воды, а высушенные —
16%. Сколько требуется свежих фруктов для приготовления 21 кг высу-
шенных фруктов?
Решение. Пусть кг фруктов требуется для приготовления кг
высушенных фруктов. Так как в фруктах содержится воды, то про-
цент сухого вещества составляет а в кг сухого вещества
равно кг.
Масса сухого вещества в высушенных кг фруктах составляет
и равна кг.
По условию задачи величина , откуда решая
уравнение, находим, что кг.
О т в е т : 252.
Задача 2. Смешав 60%−ый и 30%−ый растворы кислоты и доба-
вив 5 кг чистой воды, получили 20%−ый раствор кислоты. Если бы вместо
5 кг воды добавили 5 кг 90%−го раствора той же кислоты, то получили бы
70%−ый раствор кислоты. Сколько килограммов 60%−го раствора исполь-
зовали для получения смеси?
Решение. Пусть кг и кг — массы первого и второго растворов,
взятые при смешивании. Тогда кг — масса полученного раство-
ра, содержащего кг кислоты. Концентрация кислоты в полу-
ченном растворе 20 %, откуда получаем уравнение:
( )
Решим систему двух полученных уравнений:

130
( )
{
( )
{
Замечание. Решение можно сделать несколько проще, если заме-
тить, что из полученных уравнений следует: ( ), отку-
да . Первое уравнение принимает вид , отку-
да .
О т в е т : 2 кг.
Задача 3. Игорь и Паша красят забор за 18 часов. Паша и Володя
красят этот же забор за 20 часов, а Володя и Игорь — за 30 часов. За
сколько минут мальчики покрасят забор, работая втроём?
Решение. За один час Игорь и Паша красят 1/18 забора, Паша и
Володя красят 1/20 забора, а Володя и Игорь — за 1/30 забора. Работая
вместе, за один час два Игоря, Паши и Володи покрасили бы:
часть забора.
Тем самым, они могли бы покрасить один забор за часа.

Т.к. каждый из мальчиков был учтен два раза, в реальности Игорь,


Паша и Володя могут покрасить забор за часа минуты.
О т в е т : 864.
Задача 4. Два оператора, работая вместе, могут набрать текст газе-
ты объявлений за 8 ч. Если первый оператор будет работать 3 ч, а второй
12 ч, то они выполнят только 75% всей работы. За какое время может на-
брать весь текст каждый оператор, работая отдельно?
Решение. Пусть первый оператор может выполнить данную рабо-
ту за часов, а второй за часов. За один час первый оператор выполня-
ет часть всей работы, а второй – . Составим и решим систему уравне-
ний:

{
{
О т в е т : первый оператор за 12 ч, а второй оператор за 24 ч.

131
Задача 5. Первая труба пропускает на 2 литра воды в минуту
меньше, чем вторая. Сколько литров воды в минуту пропускает вторая
труба, если резервуар объёмом 130 литров она заполняет на 4 минуты
быстрее, чем первая труба заполняет резервуар объёмом 136 литров?
Решение. Пусть вторая труба пропускает литров воды в минуту,
тогда первая труба пропускает литра в минуту. Вторая труба запол-
няет резервуар объёмом 130 литров за минут. Поскольку первая труба
заполняет резервуар объёмом 136 литров за минут, что по условию за-
дачи на 4 минуты больше, чем получаем уравнение:

Решив данное уравнение, отбрасываем постороннее решение −6,5


и получаем, что вторая труба пропускает 10 литров в минуту.
О т в е т : 10 л.
Задача 6. Первую половину пути автомобиль проехал со скоро-
стью 42 км/ч, а вторую — со скоростью 48 км/ч. Найдите среднюю ско-
рость автомобиля на протяжении всего пути.
Решение. Обозначим условно первую половину пути за 1, и вто-
рую половину пути за 1. Соответственно, весь путь будет равен
. Время, затраченное автомобилем на прохождение первой половины
пути, равно:
часов, а вторую половину пути за часов.

Общее время в пути составило часов.


Среднюю скорость можно найти как отношение пути ко времени,
проведенном в этом пути:

О т в е т : 44,8.
Задача 7. Поезд, двигаясь равномерно со скоростью 57 км/ч, про-
езжает мимо идущего в том же направлении параллельно путям со скоро-
стью 5 км/ч пешехода за 45 секунд. Найдите длину поезда в метрах.

132
Решение. Пусть длина поезда м. Скорость поезда относительно
пешехода равна км/ч или м/c. Следовательно,
поезд проезжает мимо идущего в том же направлении параллельно путям
пешехода за сек.

Составим и решим уравнение: (м) – длина поезда.


О т в е т : 650 м.
Задача 8. Часы со стрелками показывают 8 часов 00 минут. Через
сколько минут минутная стрелка в четвертый раз поравняется с часовой?
Решение. Данная задача является задачей на движение по кругу. По
кругу движется часовая и минутная стрелки. Минутная стрелка за час про-
ходит (т.е. скорость равна в час), а часовая стрелка за час про-
ходит (скорость равна в час).

В 8:00 расстояние между стрелками . Пусть минутная стрелка в


первый раз встретится с часовой через часов, за это время минутная
стрелка пройдёт , а часовая . Зная, что минутная стрелка прой-
дёт на больше, составим и решим уравнение:

Значит, через ч стрелки встретятся первый раз. Теперь до следую-


щей встречи минутная стрелка пройдёт на больше часовой. Пусть это
произойдёт через часов, тогда получим уравнение:

И так ещё 2 раза. Получаем, что минутная стрелка в четвёртый раз по-
равняется с часовой через ч мин.
О т в е т : 240 м.

133
Задача 9. Из пункта A круговой трассы выехал велосипедист. Через
30 минут он еще не вернулся в пункт А и из пункта А следом за ним от-
правился мотоциклист. Через 10 минут после отправления он догнал вело-
сипедиста в первый раз, а еще через 30 минут после этого догнал его во
второй раз. Найдите скорость мотоциклиста, если длина трассы равна 30
км. Ответ дайте в км/ч.
Решение. Мотоциклист за минут проехал столько же, сколько вело-
сипедист за минут, следовательно, его скорость в раза
больше.
Пусть скорость велосипедиста км/ч, тогда скорость мотоциклиста
км/ч. После первой встречи за минут ( часа) велосипедист про-
ехал км, а мотоциклист км. Разница в пути составила
круг длиной км, получим и решим уравнение:

Значит, скорость велосипедиста км/ч, а скорость мотоциклиста


км/ч.
О т в е т : 80.
Задача 10. Семья состоит из мужа, жены и их дочери студентки. Если
бы зарплата мужа увеличилась вдвое, общий доход семьи вырос бы на
67%. Если бы стипендия дочери уменьшилась втрое, общий доход семьи
сократился бы на 4%. Сколько процентов от общего дохода семьи состав-
ляет зарплата жены?
Решение. Пусть доход мужа, жены и дочери (в %) соответственно рав-
ны , тогда . Зная, что если бы зарплата мужа увели-
чилась вдвое, то общий доход вырос бы на 67%, получим уравнение
. Объединим эти два уравнения в систему, получим:
{
Вычтем из второго уравнения первое и получим, что (доход
мужа в %).
Теперь первое условие объединим с условием уменьшения стипендии
дочери втрое и получим:

{ {

134
Значит, доход дочери в семье составляет . Получаем, что доход ма-
тери будет равен .
О т в е т : 27.
АНАЛИЗ ГРАФИКОВ (ИЗ ЕГЭ)
Задача 1. На рисунке изображён график функции, к которому про-
ведены касательные в четырёх точках.

Ниже указаны значения производной в данных точках. Пользуясь


графиком, поставьте в соответствие каждой точке значение производной в
ней.
ТОЧКИ ЗНАЧЕНИЯ ПРОИЗВОДНОЙ
А) 1)
Б) 2)
В) 3)
Г) 4)
Запишите в ответ цифры, расположив их в порядке, соответствую-
щем буквам:
А Б В Г

Решение. Значение производной в точке равно угловому коэффи-


циенту касательной (УКК), проведённой в этой точке. Угловой коэффици-
ент касательной:
а) положителен и меньше 1, если касательная наклонена к положи-
тельному направлению оси абсцисс под углом меньше 45°;
б) больше 1, если угол наклона больше 45°, но меньше 90°;
в) меньше −1, если угол наклона больше 90°, но меньше 135°;

135
г) больше −1 и отрицателен, если угол наклона больше 135°, но
меньше 180°.
Поэтому в точке К угловой коэффициент положителен и больше 1,
в точке L — отрицателен и меньше −1, М — отрицателен и больше −1, N
— положителен и равен 1. Таким образом, получаем соответствие А — 2,
Б — 4, В — 1 и Г — 3.
О т в е т : 2413.
Обобщение выше приведённой задачи. Пусть угол, который со-
ставляет касательная с положительным направлением оси абсцисс, равен
α, а угловой коэффициент касательной равен k. В таблице ниже приведены
соответственные числовые значения для k.
Задача 2. Установите соответствие между графиками функций и
графиками их производных.

ГРАФИКИ ФУНКЦИЙ ГРАФИКИ ПРОИЗВОДНЫХ

Решение. Известно, что производная положительна в точках, в


окрестности которых функция возрастает, отрицательна – в точках, в
окрестности которых функция убывает и нулю – в точках локальных
минимумов и максимумов функции (точках экстремума).
А) Сначала функция возрастает, т.е. производная положительна.
Этому соответствуют графики 3 и 4. В точке максимума x=0 производная
должна быть равна 0 – это график под номером 4.
Б) В начале функция убывает, следовательно, ее производная
начинается в отрицательной области – это графики 1 и 2. Точка минимума
x=-1, в которой производная равна 0, имеем график под номером 2.

136
В) Производная должна идти из отрицательной области в положи-
тельную и в точке x=0 быть равной 0. Это график под номером 1.
Г) Производная идет из положительной области в отрицательную
и в точке x=3 равна 0 – это график под номером 3.
О т в е т : 4213.
Обратите внимание! Если график производной y= f′(x) представ-
ляет собой прямую, параллельную оси ox (y=b, где b – число), то функция
y=f(x) — линейная. Она является возрастающей, если b>0, убывающей,
если b<0, и постоянной, если b=0.
Также можно использовать таблицу с k ниже, где k будет точкой,
через которую проходит прямая производной по y.

Задача 3. На рисунке изображен график производной функ-


ции f(x), определенной на интервале (−16; 4). Найдите количество точек
экстремума функции f(x) на отрезке [−14; 2].

Решение. Точки экстремума соответствуют точкам смены зна-


ка производной – изображенным на графике нулям производной.
Производная обращается в нуль в точках −13, −11, −9, −7. На от-
резке [−14; 2] функция имеет 4 точки экстремума.
О т в е т : 4.

137
ЗАДАЧИ НА СМЕКАЛКУ (ИЗ ЕГЭ)
Задача 1. Из книги выпало несколько идущих подряд листов. Но-
мер последней страницы перед выпавшими листами — 498, номер первой
страницы после выпавших листов записывается теми же цифрами, но в
другом порядке. Сколько листов выпало?
Решение. Обратите внимание, в задании спрашивают число вы-
павших листов, а не страниц. Как известно, на листе две страницы. Отсюда
следует, что номер первой страницы после выпавших листов будет нечет-
ным, так как с нечетных номеров начинается нумерация страниц на ли-
стах.
Далее, в задании сказано, что этот номер страницы можно запи-
сать теми же цифрами 498, поэтому, имеем такие варианты: 489, 948, 984,
849, 894. Среди них только два с нечетными номерами – 489 и 849. Но, так
как 489 < 498, то оно не подходит. Остается только 849. Значит, число вы-
павших листов, равно: ( ) .
Здесь –1 приводит воедино нумерацию листов, так как 498 – это
вторая страница листа, а 849 – первая страница. А для подсчета листов
нужно вычитать из первых страниц.
О т в е т : 175.
Задача 2. Каждую секунду бактерия делится на две новые бакте-
рии. Известно, что весь объём одного стакана бактерии заполняют за 1 час.
За сколько секунд стакан будет заполнен бактериями наполовину?
Решение. Заметим, что каждую секунду в стакане становится в два
раза больше бактерий. То есть если в какой-то момент бактериями запол-
нена половина стакана, то через секунду будет заполнен весь стакан. Т.о.,
полстакана будет заполнено через 59 минут и 59 секунд то есть через 3599
секунд.
О т в е т : 3599.
Задача 3. На палке отмечены поперечные линии красного, жёлто-
го и зелёного цвета. Если распилить палку по красным линиям, получится
15 кусков, если по жёлтым — 5 кусков, а если по зелёным — 7 кусков.
Сколько кусков получится, если распилить палку по линиям всех трёх цве-
тов?

138
Решение. Если распилить палку по красным линиям, то получится
15 кусков, следовательно, линий — 14. Если распилить палку по жёлтым
— 5 кусков, следовательно, линий — 4. Если распилить по зеленым — 7
кусков, линий — 6. Всего линий: 14 + 4 + 6 = 24 линии, следовательно,
кусков будет 25.
О т в е т : 25.
Задача 4. На поверхности глобуса фломастером проведены 17 па-
раллелей и 24 меридиана. На сколько частей проведённые линии раздели-
ли поверхность глобуса?
Меридиан — это дуга окружности, соединяющая Северный и
Южный полюсы. Параллель — это окружность, лежащая в плоскости, па-
раллельной плоскости экватора.
Решение. Семнадцать параллелей разделили глобус на 18 частей,
следовательно, 18 · 24 = 432 — на столько частей разделят глобус 17 па-
раллелей и 24 меридиана.
О т в е т : 432.
Задача 5. Семь столбов соединены между собой проводами так,
что от каждого столба отходит ровно 4 провода. Сколько всего проводов
между этими семью столбами?
Решение. Каждый столб связан с другим столбом и всего соедине-
ний . Но два столба связаны
друг с другом одним проводом, поэтому проводов между ними в 2 раза
меньше, т.е. .
Ответ: 14.
Задача 6. Из десяти стран семь подписали договор о дружбе ровно
с тремя другими странами, а каждая из оставшихся трёх — ровно с семью.
Сколько всего было подписано договоров?
Решение. Всего было подписей .
У одного договора две подписи, значит, всего было подписано
договор.
О т в е т : 21.
Задача 7. Миша, Коля и Лёша играют в настольный теннис: игрок,
проигравший партию, уступает место игроку, не участвовавшему в ней. В

139
итоге оказалось, что Миша сыграл 12 партий, а Коля – 25. Сколько партий
сыграл Лёша?
Решение. Больше всех сыграл Коля (25 партий), значит, всего бы-
ло сыграно не менее 25 партий. Из первых двух партий подряд хотя бы в
одном должен участвовать Миша, значит, партий было не более
. Получается, что всего было сыграно 25 партий и Коля участвовал
в каждой из них (т.к. он сыграл 25 партий по условию). В 12 партиях он
встречался с Мишей, а в партиях с Лёшей.
О т в е т : 13.
Задача 8. Клетки таблицы 6 × 5 раскрашены в чёрный и белый
цвета. Пар соседних клеток разного цвета 26, пар соседних клеток чёрного
цвета всего 6. Сколько пар соседних клеток белого цвета?
Решение. В одной строке 6 клеток, а значит, 5 пар соседних клеток
(клетка 1 и 2, 2 и 3, 3 и 4, 4 и 5, 5 и 6). По условию, всего 5 строк, значит, в
этих 5 строках пар соседних клеток.
В одном столбце 5 клеток, а значит, 4 пары соседних клеток (клет-
ка 1 и 2, 2 и 3, 3 и 4, 4 и 5). По условию, всего 6 строк, значит, в этих 6
строках пары соседних клеток.
Получается, что всего в таблица 6 × 5 пар клеток.
Т.к. по условию мы знаем, что таблица раскрашена в чёрный и белые цве-
та, то она состоит из пар соседних клеток разного цвета (по условию 26
пар), чёрного цвета (6 пар) и белого цвета (x пар).
Выше мы выяснили, что всего пар соседних клеток 49, значит, со-
ставим уравнение по цветам:
;
.
Всего получилось 17 пар соседних клеток белого цвета.
О т в е т : 17.
Задача 9. В корзине лежит 40 грибов: рыжики и грузди. Известно,
что среди любых 17 грибов имеется хотя бы один рыжик, а среди любых
25 грибов хотя бы один груздь. Сколько рыжиков в корзине?
Решение. Груздей максимум 16 (иначе можно было бы взять 17
груздей и условие бы не выполнилось). Рыжиков максимум 24 (иначе

140
можно было бы взять 25 груздей в нарушение условия). Известно, что в
корзине всего 40 грибов. Поэтому груздей ровно 16, а рыжиков ровно 24.
О т в е т : 24.
Задача 10. Саша пригласил Петю в гости, сказав, что живёт в
седьмом подъезде в квартире № 462, а этаж сказать забыл. Подойдя к до-
му, Петя обнаружил, что дом семиэтажный. На каком этаже живёт Саша?
(На всех этажах число квартир одинаково, номера квартир в доме начина-
ются с единицы.)
Решение. Поскольку в первых 7 подъездах не меньше 462 квартир,
в каждом подъезде не меньше 462 : 7 = 66 квартир. Следовательно, на каж-
дом из 7 этажей в подъезде не меньше 9 квартир.
Пусть на каждой лестничной площадке по 9 квартир. Тогда в пер-
вых семи подъездах всего 9 · 7 · 7 = 441 квартира, и квартира 462 окажется
в восьмом подъезде, что противоречит условию.
Пусть на каждой площадке по 10 квартир. Тогда в первых семи
подъездах 10 · 7 · 7 = 490 квартир, а в первых шести — 420. Следователь-
но, квартира 462 находится в седьмом подъезде. Она в нем 42-ая по счету,
поскольку на этаже по 10 квартир, она расположена на пятом этаже. Если
бы на каждой площадке было по 11 квартир, то в первых шести подъездах
оказалось бы 11 · 7 · 6 = 462 квартиры, то есть 462 квартира в шестом
подъезде, что противоречит условию. Значит, Саша живёт на пятом этаже.
Ответ: 5
Задача 11. Сколькими способами можно поставить в ряд два оди-
наковых красных кубика, три одинаковых зелёных кубика и один синий
кубик?
Решение. Занумеруем все кубики от одного до шести. Пока не
учитываем, что в нашем наборе есть кубики одинакового цвета. На первое
место можно поставить кубик шестью способами, на второе — пятью, на
третье — четырьмя и так далее. Получаем, что всего возможностей рас-
становки кубиков .
Теперь учтём, что перестановка, например, двух красных кубиков
не даёт нового способа расстановки кубиков. В любом полученном выше
наборе можно переставить красные кубики местами, то есть число расста-
новок уменьшится в два раза.

141
С зелёными кубиками аналогично. Зелёных кубиков три, поэтому
в любом полученном выше наборе можно переставлять их, не получая но-
вых способов расстановки кубиков. Таких перестановок зелёных куби-
ков .
Следовательно, искомое число способов равно: .
О т в е т : 60.
Задача 12. В бак объёмом 38 литров каждый час, начиная с 12 ча-
сов, наливают полное ведро воды объёмом 8 литров. Но в днище бака есть
небольшая щель, и из неё за час вытекает 3 литра. В какой момент времени
(в часах) бак будет заполнен полностью.
Решение. К концу каждого часа объём воды в баке увеличивается
на 8 − 3 = 5 литров. Через 6 часов, то есть в 18 часов, в баке будет 30 лит-
ров воды. В 18 часов в бак дольют 8 литров воды и объём воды в баке ста-
нет равным 38 литров.
О т в е т : 18.
Задача 13 (авторское решение). В обменном пункте можно совер-
шить 1-у из 2-ух операций:
• за 2 золотых монеты получить 3 серебряных и одну медную;
• за 5 серебряных монет получить 3 золотых и одну медную.
У Николая были только серебряные монеты. После нескольких посе-
щений обменного пункта серебряных монет у него стало меньше, золотых
не появилось, зато появилось 50 медных. На сколько уменьшилось количе-
ство серебряных монет у Николая?
Решение. Обозначим золотые монеты буквой , серебрянные – , а
медные – . Опираясь на условие задачи, составим и решим систему урав-
нений:
{
Т.к. у Николая не появилось золотых монет, то нам нужно, чтобы
золотые монеты сократились в ходе решения системы. Для этого мы пере-
несём все составляющие уравнений влево, а справа останется 0. Умножим
первое уравнение на 3, а второе – на 2 и сложим их:
{ {

142
В результате сложения данных уравнений системы получим:
Т.к. медных стало 50 штук, то и серебряных уменьшилось на 10
штук соответственно.
О т в е т : 10.
Задача 14. Маша и Медведь съели 160 печений и банку варенья,
начав и закончив одновременно. Сначала Маша ела варенье, а Медведь –
печенье, но в какой-то момент они поменялись. Медведь и то, и другое ест
в три раза быстрее Маши. Сколько печений съел Медведь, если варенье
они съели поровну?
Решение. Медведь съел свою половину банки варенья в 3 раза
быстрее, чем Маша, значит, у него еще осталось в 3 раза больше времени
на кушанье печенья.
Т.к. Медведь ест печенье в 3 раза быстрее, чем Маша и еще у него
осталось в 3 раза больше времени (он съел в 3 раза быстрее свою половину
банки варенья), то он съедает в 3 3=9 раз больше печений, чем Маша (9
печений съедает Медведь, в то время как Маша только 1 печенье).
Получается, что в отношении 9:1 едят Медведь и Маша печенье.
Всего получается 10 долей, значит, 1 доля равна 160:10=16. В итоге, Мед-
ведь съел 16 9=144 печений.
О т в е т : 144.
Задача 15 (авторское решение). Список заданий викторины состо-
ял из 33 вопросов. За каждый правильный ответ ученик получал 7 очков,
за неправильный ответ с него списывали 11 очков, а при отсутствии ответа
давали 0 очков. Сколько верных ответов дал ученик, набравший 84 очка,
если известно, что по крайней мере один раз он ошибся?
Решение. Пусть ученик дал правильных ответов и – непра-
вильных ответов, – количество вопросов на которые не было дано отве-
тов. Зная, что ученик всего набрал 84 очка, составим уравнение и выразим
из него неправильные ответы (переменную , т.к. он хотя бы 1 раз ошиб-
ся):

Далее, используя метод подбора и признак делимости на 11, под-


бираем такой числитель, который разделится на 11, чтобы вышло целое

143
число (т.к. это количество ответов). Помним, что искомое число не
больше 33. Т.о., находим, что , т.к. – целое число.
О т в е т : 23.
Задача 16. На кольцевой дороге расположены четыре бензоколон-
ки: A, B, C и D. Расстояние между A и B — 50 км, A и C — 40 км, C и D —
25 км, D и A — 35 км (все расстояния измеряются вдоль кольцевой дороги
в кратчайшую сторону). Найдите расстояние между B и C.
Решение. Расположим А, В, C, D вдоль кольце-
вой дороги по очереди так, чтобы расстояния соответ-
ствовали данным в условии. Всё хорошо, кроме расстоя-
ния между D и A. Чтобы оно было таким, каким нужно,
подвинем D и поставим между B и A нужным образом.
Тогда между B и D будет 15 км. А между B и С —10 км.
О т в е т : 10.
Задача 17 (авторское решение). На прилавке цветочного магазина
стоят 3 вазы с розами: чёрная, зелёная и оранжевая. Слева от чёрной вазы
32 розы, справа от оранжевой вазы 9 роз. Всего в вазах 37 роз. Сколько роз
в зелёной вазе?
Решение. Исходя из условия задачи (слева от чёрной (Ч) вазы 32
розы, а справа от оранжевой (О) – 9 роз) можно сделать вывод, что чёрная
ваза стоит не левее всех остальных, а оранжевая – не правее всех осталь-
ных. Значит, оранжевая ваза может стоять на 1 или 2 месте, а чёрная мо-
жет стоять на 2 или 3 месте. Зелёная (З) ваза может стоять на любом месте.
Далее необходимо рассмотреть такие случаи расположения ваз:
ОЗЧ, ОЧЗ и ЗОЧ. Рассмотрев все случаи, убедимся, что подходящий нам
случай – ОЗЧ.

О т в е т : 4.

144
Задача 18. Три луча, выходящие из одной точки, разбивают плос-
кость на 3 разных угла, измеряемых целым числом градусов. Наибольший
угол в 4 раза больше наименьшего. Сколько значений может принимать
величина среднего угла?
Решение. Обозначим через градусов значение наименьшего угла.
Тогда наибольший будет равен градусов. Сумма всех углов, получае-
мых таких разбиением, равна 360° (круг). Получаем следующее равенство:

где – размер среднего угла (в градусах). При этом, . Отсюда


следует, что наименьшее значение малого угла

откуда получаем, что . А наибольшее его значение


. Таким образом, общее число вариантов среднего угла, равно:

(здесь учитывает, что отсчет начинается с угла 41° и заканчивается


углом 59°).
О т в е т : 19.
Задача 19 (авторское решение). Прямоугольник двумя прямоли-
нейными разрезами разбит на четыре малых прямоугольника (см. рису-
нок). Периметры трёх из них, начиная с левого верхнего и далее по часо-
вой стрелке, равны 17, 12 и 13. Найдите периметр четвертого прямоуголь-
ника.

Решение. Периметр искомого прямоугольника можно вычислить,


сложив известные периметры противолежащих прямоугольников и отняв
из этой суммы периметр прямоугольника, находящегося напротив неиз-
вестного: Т.к. мы сложили все стороны противо-
лежащих прямоугольников, мы получили по 2 лишние стороны для неиз-

145
вестного прямоугольника, которые нужно отнять (а эти пары лишних сто-
рон и дают в сумме периметр прямоугольника, находящегося напротив
неизвестного), именно поэтому мы потом и отнимаем данный периметр.
О т в е т : 18.
Задача 20. В доме всего пятнадцать квартир с номерами от 1 до
15. В каждой квартире живёт не менее одного и не более трёх человек. В
квартирах с 1-й по 12-ю включительно живёт суммарно 14 человек, а в
квартирах с 11-й по 15-ю включительно живёт суммарно 13 человек.
Сколько всего человек живёт в этом доме?
Решение. В квартире могут жить один, два или три человека. В
квартирах с 1-й по 12-ю включительно живёт суммарно 14 человек, следо-
вательно, в 10 квартирах живёт по одному человеку, а в оставшихся двух
квартирах живёт суммарно 4 человека. В квартирах с 11-й по 15-ю вклю-
чительно живёт суммарно 13 человек, следовательно, в трёх квартирах
живёт по три человека, а в оставшихся двух живёт суммарно 4 человека.
Рассмотренные множества квартир пересекаются по квартирам 11 и 12,
значит, именно в квартирах 11 и 12 в сумме живёт 4 человека. Таким обра-
зом, получаем, что всего в доме живёт 10 · 1 + 4 + 3 · 3 = 23 человек.
О т в е т : 23.
Задача 21. Взяли несколько досок и распилили их. Всего сделали
11 поперечных распилов, в итоге получилось 16 кусков. Сколько досок
взяли?
Решение. Каждый поперечный распил добавляет один кусок к уже
имеющимся, следовательно, изначально было 16 − 11 = 5 досок.
О т в е т : 5.
Задача 22 (авторское решение). Петя меняет маленькие фишки на
большие. За один обмен он получает 3 большие фишки, отдав 10 малень-
ких. До обменов у Пети было 100 фишек (среди них были и большие, и
маленькие), а после стало 65. Сколько обменов он совершил?
Решение. За один обмен количество фишек у Пети уменьшается на
10 − 3 = 7 штук. Зная, что всего у него было 100 фишек, а осталось – 65,
составим и решим уравнение, приняв за – количество обменов:
– .
О т в е т : 5.

146
Задача 23. Если бы каждый из двух множителей увеличили на 1,
их произведение увеличилось бы на 11. На сколько увеличится произведе-
ние этих множителей, если каждый из них увеличить на 2?
Решение. Пусть – соответственно первое и второе число. При
увеличении каждого из множителей на 1 их произведение увеличивается
на 11:
( )( )
Найдём на сколько увеличится произведение этих множителей при
увеличении каждого из них на 2:
( )( ) ( )
Таким образом, при увеличении каждого из множителей на 2, их про-
изведение увеличивается на 24.
О т в е т : 24.
Задача 24. В таблице три столбца и несколько строк. В каждую
клетку таблицы вписали по натуральному числу так, что сумма всех чисел
в первом столбце равна 72, во втором — 81, в третьем — 91, а сумма чисел
в каждой строке больше 13, но меньше 16. Сколько всего строк в таблице?
Решение. Сумма всех чисел в таблице равна 72 + 81 + 91 = 244.
Сумма чисел в каждой строке может быть равна 14 или 15. В таблице не
может быть больше, чем строк. И не может быть мень-
ше строк. Следовательно, в данной таблице ровно 17 строк.
О т в е т : 17.

147
СОДЕРЖАНИЕ

АЛГЕБРА

 ТЕМА 1. НАТУРАЛЬНЫЕ ЧИСЛА И ДЕЙСТВИЯ НАД НИМИ ....... 3


 ТЕМА 2. ОБЫКНОВЕННЫЕ ДРОБИ, ДЕСЯТИЧНЫЕ ДРОБИ,
СМЕШАННЫЕ ЧИСЛА И ДЕЙСТВИЯ НАД НИМИ ......................... 4
 ТЕМА 3. ЦЕЛЫЕ И РАЦИОНАЛЬНЫЕ ЧИСЛА, ДЕЙСТВИЯ НАД
НИМИ. ПРОПОРЦИИ ........................................................................ 7
 ТЕМА 4. ПРОЦЕНТЫ. ФОРМУЛА ПРОИЗВОДИТЕЛЬНОСТИ В
ЗАДАЧАХ .......................................................................................... 9
 ТЕМА 5. СТЕПЕНЬ С НАТУРАЛЬНЫМ И ЦЕЛЫМ ПОКАЗАТЕЛЕМ
( )................................................................................................. 9
 ТЕМА 6. ОДНОЧЛЕНЫ И МНОГОЧЛЕНЫ ..................................... 10
 ТЕМА 7. ЦЕЛЫЕ И ДРОБНЫЕ ВЫРАЖЕНИЯ. АЛГЕБРАИЧЕСКИЕ
ДРОБИ .............................................................................................. 12
 ТЕМА 8. АРИФМЕТИЧЕСКИЙ КВАДРАТНЫЙ КОРЕНЬ И ЕГО
СВОЙСТВА ...................................................................................... 12
 ТЕМА 9. УРАВНЕНИЯ, НЕРАВЕНСТВА И ИХ СИСТЕМЫ ........... 13
 ТЕМА 10. ФУНКЦИИ И ИХ ОСНОВНЫЕ СВОЙСТВА .................. 17
 ТЕМА 11. ЛИНЕЙНАЯ ФУНКЦИЯ И ЛИНЕЙНЫЕ УРАВНЕНИЯ . 24
 ТЕМА 12. КВАДРАТИЧНАЯ ФУНКЦИЯ ........................................ 25
 ТЕМА 13. РАЦИОНАЛЬНЫЕ УРАВНЕНИЯ И НЕРАВЕНСТВА .... 27
 ТЕМА 14. АРИФМЕТИЧЕСКАЯ И ГЕОМЕТРИЧЕСКАЯ ПРОГРЕС-
СИИ .................................................................................................. 29
 ТЕМА 15. СИНУС, КОСИНУС, ТАНГЕНС И КОТАНГЕНС. РАДИ-
АННАЯ МЕРА .................................................................................. 32

148
 ТЕМА 16. ОСНОВНЫЕ ТРИГОНОМЕТРИЧЕСКИЕ ФОРМУЛЫ .... 35
 ТЕМА 17. ТРИГОНОМЕТРИЧЕСКИЕ И ОБРАТНЫЕ ФУНКЦИИ .. 36
 ТЕМА 18. ТРИГОНОМЕТРИЧЕСКИЕ УРАВНЕНИЯ И НЕРАВЕН-
СТВА ................................................................................................ 39
 ТЕМА 19. УРАВНЕНИЯ И НЕРАВЕНСТВА С МОДУЛЕМ ............ 50
 ТЕМА 20. КОРЕНЬ n-й СТЕПЕНИ. СТЕПЕНЬ С РАЦИОНАЛЬНЫМ
ПОКАЗАТЕЛЕМ ....................................................................................... 50
 ТЕМА 21. СТЕПЕННАЯ ФУНКЦИЯ..................................................... 51
 ТЕМА 22. ИРРАЦИОНАЛЬНЫЕ УРАВНЕНИЯ И НЕРАВЕНСТВА 53
 ТЕМА 23. ПОКАЗАТЕЛЬНАЯ ФУНКЦИЯ И ПОКАЗАТЕЛЬНЫЕ
УРАВНЕНИЯ ................................................................................... 55
 ТЕМА 24. ЛОГАРИФМЫ. ЛОГАРИФМИЧЕСКАЯ ФУНКЦИЯ. ЛО-
ГАРИФМИЧЕСКИЕ УРАВНЕНИЯ И НЕРАВЕНСТВА ................... 56
 ТЕМА 25. ПРОИЗВОДНАЯ ФУНКЦИИ. ДИФФЕРЕНЦИРОВАНИЕ.
НАХОЖДЕНИЕ ЭКСТРЕМУМА ФУНКЦИИ .................................. 58
 ТЕМА 26. ПЕРВООБРАЗНАЯ. НЕОПРЕДЕЛЁННЫЙ И ОПРЕДЕ-
ЛЁННЫЙ ИНТЕГРАЛЫ ................................................................... 62
 ТЕМА 27. НАЧАЛА ТЕОРИИ ВЕРОЯТНОСТИ............................... 66
 ТЕМА 28. ЭЛЕМЕНТЫ КОМБИНАТОРИКИ .................................. 68

ГЕОМЕТРИЯ

 ТЕМА 1. ПРОСТЕЙШИЕ ГЕОМЕТРИЧЕСКИЕ ФИГУРЫ И ИХ


СВОЙСТВА ...................................................................................... 71
 ТЕМА 2. ТРЕУГОЛЬНИК. ПРИЗНАКИ РАВЕНСТВА ТРЕУГОЛЬ-
НИКОВ. ВИДЫ ТРЕУГОЛЬНИКОВ ................................................. 75

149
 ТЕМА 3. СУММА УГЛОВ ТРЕУГОЛЬНИКА. НЕРАВЕНСТВО ТРЕ-
УГОЛЬНИКА ................................................................................... 79
 ТЕМА 4. ПОДОБИЕ ТРЕУГОЛЬНИКОВ. СВОЙСТВА ПОДОБНЫХ
ТРЕУГОЛЬНИКОВ. ТЕОРЕМА ПИФАГОРА ....................................... 80
 ТЕМА 5. ПЛОЩАДЬ ТРЕУГОЛЬНИКА. ПОНЯТИЕ ПЛОЩАДИ .... 81
 ТЕМА 6. ОКРУЖНОСТЬ И КРУГ. КАСАТЕЛЬНАЯ К ОКРУЖНО-
СТИ ............................................................................................................ 82
 ТЕМА 7. ЦЕНТРАЛЬНЫЙ И ВПИСАННЫЙ УГЛЫ ........................ 83
 ТЕМА 8. ДЛИНА КРУГА И ПЛОЩАДЬ КРУГА ............................. 84
 ТЕМА 9. ВПИСАННЫЕ И ОПИСАННЫЕ ОКРУЖНОСТИ ............. 85
 ТЕМА 10. ТЕОРЕМА КОСИНУСОВ И СИНУСОВ ......................... 86
 ТЕМА 11. ЧЕТЫРЕХУГОЛЬНИКИ: ПАРАЛЛЕЛОГРАММ, ПРЯМО-
УГОЛЬНИК, РОМБ, КВАДРАТ, ТРАПЕЦИЯ ................................... 88
 ТЕМА 12. ПЛОЩАДИ ЧЕТЫРЕХУГОЛЬНИКОВ ........................... 91
 ТЕМА 13. ЛОМАНАЯ. МНОГОУГОЛЬНИКИ ................................. 92
 ТЕМА 14. ДЕКАРТОВЫ КООРДИНАТЫ НА ПЛОСКОСТИ .......... 95
 ТЕМА 15. ВЕКТОРЫ НА ПЛОСКОСТИ .......................................... 97
 ТЕМА 16. ВВЕДЕНИЕ В СТЕРЕОМЕТРИЮ. АКСИОМЫ СТЕРЕО-
МЕТРИИ.......................................................................................... 100
 ТЕМА 17. МНОГОГРАННИКИ....................................................... 101
 ТЕМА 18. ПИРАМИДА. ЕЁ ВИДЫ И СВОЙСТВА......................... 103
 ТЕМА 19. ПЛОЩАДИ И ОБЪЕМЫ МНОГОГРАННИКОВ ............ 105
 ТЕМА 20. ТЕЛА ВРАЩЕНИЯ: ЦИЛИНДР И КОНУС. ШАР И СФЕ-
РА. ПЛОЩАДИ И ОБЪЁМЫ ........................................................... 106
 ТЕМА 21. ДЕКАРТОВЫ КООРДИНАТЫ В ПРОСТРАНСТВЕ ...... 108
 ПРИЛОЖЕНИЕ ............................................................................. 110

150
151

Вам также может понравиться